Slot System
Featured Buckets
Featured Buckets Admin

Myth of the Month: Is Contrast-Induced Acute Kidney Injury Real?

Article Type
Changed
Mon, 05/13/2024 - 09:24

A 59-year-old man presents with abdominal pain. He has a history of small bowel obstruction and diverticulitis. His medical history includes chronic kidney disease (CKD; baseline creatinine, 1.8 mg/dL), hypertension, type 2 diabetes, and depression. He had a colectomy 6 years ago for colon cancer.

He takes the following medications: Semaglutide (1 mg weekly), amlodipine (5 mg once daily), and escitalopram (10 mg once daily). On physical exam his blood pressure is 130/80 mm Hg, his pulse is 90, and his temperature is 37.2 degrees C. He has normal bowel sounds but guarding in the right lower quadrant.

Paauw_Doug_SEATTLE_2019_web.jpg
Dr. Douglas S. Paauw


His hemoglobin is 14 g/dL, his blood sodium is 136 mEq/L, his blood potassium is 4.0 mmol/L, his BUN is 26 mg/dL, and his creatinine is 1.9 mg/dL. His kidney, ureter, bladder x-ray is unremarkable.
 

What imaging would you recommend?

A) CT without contrast

B) CT with contrast

C) MRI

D) Abdominal ultrasound

This patient has several potential causes for his abdominal pain that imaging may clarify. I think a contrast CT scan would be the most likely to provide helpful information. It is likely that if it were ordered, there may be hesitation by the radiologist to perform the scan with contrast because of the patient’s CKD.

Concern for contrast-induced kidney injury has limited diagnostic testing for many years. How strong is the evidence for contrast-induced kidney injury, and should we avoid testing that requires contrast in patients with CKD? McDonald and colleagues performed a meta-analysis with 13 studies meeting inclusion criteria, involving 25,950 patients.1 They found no increased risk of acute kidney injury (AKI) in patients who received contrast medium compared with those who did not receive contrast; relative risk of AKI for those receiving contrast was 0.79 (confidence interval: 0.62-1.02). Importantly, there was no difference in AKI in patients with diabetes or CKD.

Ehmann et al. looked at renal outcomes in patients who received IV contrast when they presented to an emergency department with AKI.2 They found that in patients with AKI, receiving contrast was not associated with persistent AKI at hospital discharge. Hinson and colleagues looked at patients arriving at the emergency department and needing imaging.3 They did a retrospective, cohort analysis of 17,934 patients who had CT with contrast, CT with no contrast, or no CT. Contrast administration was not associated with increased incidence of AKI (odds ratio, 0.96, CI: 0.85-1.08).

Aycock et al. did a meta-analysis of AKI after CT scanning, including 28 studies involving 107,335 patients.4 They found that compared with noncontrast CT, CT scanning with contrast was not associated with AKI (OR, 0.94, CI: 0.83-1.07). Elias and Aronson looked at the risk of AKI after contrast in patients receiving CT scans compared with those who received ventilation/perfusion scans to evaluate for pulmonary embolism.5 There were 44 AKI events (4.5%) in patients exposed to contrast media and 33 events (3.4%) in patients not exposed to contrast media (risk difference: 1.1%, 95% CI: -0.6% to 2.9%; OR, 1.39, CI: 0.86-2.26; P = .18).

Despite multiple studies showing no increased risk, there is still a concern that contrast can cause AKI.6 Animal models have shown iodinated contrast can have a deleterious effect on mitochondria and membrane function.6 Criticisms of the retrospective nature of many of the studies I have shared, and the lack of randomized, controlled trials are that there may be bias in these studies, as the highest-risk patients are the ones most likely not to receive contrast. In a joint guideline from the American College of Radiology and the National Kidney Foundation, this statement was made: “The risk of acute kidney injury developing in patients with reduced kidney function following exposure to intravenous iodinated contrast media has been overstated.”7 Their recommendation was to give contrast if needed in patients with glomerular filtration rates (GFRs) greater than 30.



Myth: Contrast-induced renal injury is a concern.

Clinical impact: For CT scanning, it is OK to give contrast when needed. A conservative cutoff for contrast use would be a GFR less than 30.
 

Dr. Paauw is professor of medicine in the Division of General Internal Medicine at the University of Washington, Seattle, and he serves as third-year medical student clerkship director at the University of Washington. Contact Dr. Paauw at dpaauw@uw.edu.

References

1. McDonald JS et al. Radiology. 2013:267:119-128.

2. Ehmann MR et al. Intensive Care Med. 2023:49(2):205-215.

3. Hinson JS et al. Ann Emerg Med. 2017;69(5):577-586.

4. Aycock RD et al. Ann Emerg Med. 2018 Jan;71(1):44-53.

5. Elias A, Aronson D. Thromb Haemost. 2021 Jun;121(6):800-807.

6. Weisbord SD, du Cheryon D. Intensive Care Med. 2018;44(1):107-109.

7. Davenport MS et al. Radiology. 2020;294(3):660-668.

Publications
Topics
Sections

A 59-year-old man presents with abdominal pain. He has a history of small bowel obstruction and diverticulitis. His medical history includes chronic kidney disease (CKD; baseline creatinine, 1.8 mg/dL), hypertension, type 2 diabetes, and depression. He had a colectomy 6 years ago for colon cancer.

He takes the following medications: Semaglutide (1 mg weekly), amlodipine (5 mg once daily), and escitalopram (10 mg once daily). On physical exam his blood pressure is 130/80 mm Hg, his pulse is 90, and his temperature is 37.2 degrees C. He has normal bowel sounds but guarding in the right lower quadrant.

Paauw_Doug_SEATTLE_2019_web.jpg
Dr. Douglas S. Paauw


His hemoglobin is 14 g/dL, his blood sodium is 136 mEq/L, his blood potassium is 4.0 mmol/L, his BUN is 26 mg/dL, and his creatinine is 1.9 mg/dL. His kidney, ureter, bladder x-ray is unremarkable.
 

What imaging would you recommend?

A) CT without contrast

B) CT with contrast

C) MRI

D) Abdominal ultrasound

This patient has several potential causes for his abdominal pain that imaging may clarify. I think a contrast CT scan would be the most likely to provide helpful information. It is likely that if it were ordered, there may be hesitation by the radiologist to perform the scan with contrast because of the patient’s CKD.

Concern for contrast-induced kidney injury has limited diagnostic testing for many years. How strong is the evidence for contrast-induced kidney injury, and should we avoid testing that requires contrast in patients with CKD? McDonald and colleagues performed a meta-analysis with 13 studies meeting inclusion criteria, involving 25,950 patients.1 They found no increased risk of acute kidney injury (AKI) in patients who received contrast medium compared with those who did not receive contrast; relative risk of AKI for those receiving contrast was 0.79 (confidence interval: 0.62-1.02). Importantly, there was no difference in AKI in patients with diabetes or CKD.

Ehmann et al. looked at renal outcomes in patients who received IV contrast when they presented to an emergency department with AKI.2 They found that in patients with AKI, receiving contrast was not associated with persistent AKI at hospital discharge. Hinson and colleagues looked at patients arriving at the emergency department and needing imaging.3 They did a retrospective, cohort analysis of 17,934 patients who had CT with contrast, CT with no contrast, or no CT. Contrast administration was not associated with increased incidence of AKI (odds ratio, 0.96, CI: 0.85-1.08).

Aycock et al. did a meta-analysis of AKI after CT scanning, including 28 studies involving 107,335 patients.4 They found that compared with noncontrast CT, CT scanning with contrast was not associated with AKI (OR, 0.94, CI: 0.83-1.07). Elias and Aronson looked at the risk of AKI after contrast in patients receiving CT scans compared with those who received ventilation/perfusion scans to evaluate for pulmonary embolism.5 There were 44 AKI events (4.5%) in patients exposed to contrast media and 33 events (3.4%) in patients not exposed to contrast media (risk difference: 1.1%, 95% CI: -0.6% to 2.9%; OR, 1.39, CI: 0.86-2.26; P = .18).

Despite multiple studies showing no increased risk, there is still a concern that contrast can cause AKI.6 Animal models have shown iodinated contrast can have a deleterious effect on mitochondria and membrane function.6 Criticisms of the retrospective nature of many of the studies I have shared, and the lack of randomized, controlled trials are that there may be bias in these studies, as the highest-risk patients are the ones most likely not to receive contrast. In a joint guideline from the American College of Radiology and the National Kidney Foundation, this statement was made: “The risk of acute kidney injury developing in patients with reduced kidney function following exposure to intravenous iodinated contrast media has been overstated.”7 Their recommendation was to give contrast if needed in patients with glomerular filtration rates (GFRs) greater than 30.



Myth: Contrast-induced renal injury is a concern.

Clinical impact: For CT scanning, it is OK to give contrast when needed. A conservative cutoff for contrast use would be a GFR less than 30.
 

Dr. Paauw is professor of medicine in the Division of General Internal Medicine at the University of Washington, Seattle, and he serves as third-year medical student clerkship director at the University of Washington. Contact Dr. Paauw at dpaauw@uw.edu.

References

1. McDonald JS et al. Radiology. 2013:267:119-128.

2. Ehmann MR et al. Intensive Care Med. 2023:49(2):205-215.

3. Hinson JS et al. Ann Emerg Med. 2017;69(5):577-586.

4. Aycock RD et al. Ann Emerg Med. 2018 Jan;71(1):44-53.

5. Elias A, Aronson D. Thromb Haemost. 2021 Jun;121(6):800-807.

6. Weisbord SD, du Cheryon D. Intensive Care Med. 2018;44(1):107-109.

7. Davenport MS et al. Radiology. 2020;294(3):660-668.

A 59-year-old man presents with abdominal pain. He has a history of small bowel obstruction and diverticulitis. His medical history includes chronic kidney disease (CKD; baseline creatinine, 1.8 mg/dL), hypertension, type 2 diabetes, and depression. He had a colectomy 6 years ago for colon cancer.

He takes the following medications: Semaglutide (1 mg weekly), amlodipine (5 mg once daily), and escitalopram (10 mg once daily). On physical exam his blood pressure is 130/80 mm Hg, his pulse is 90, and his temperature is 37.2 degrees C. He has normal bowel sounds but guarding in the right lower quadrant.

Paauw_Doug_SEATTLE_2019_web.jpg
Dr. Douglas S. Paauw


His hemoglobin is 14 g/dL, his blood sodium is 136 mEq/L, his blood potassium is 4.0 mmol/L, his BUN is 26 mg/dL, and his creatinine is 1.9 mg/dL. His kidney, ureter, bladder x-ray is unremarkable.
 

What imaging would you recommend?

A) CT without contrast

B) CT with contrast

C) MRI

D) Abdominal ultrasound

This patient has several potential causes for his abdominal pain that imaging may clarify. I think a contrast CT scan would be the most likely to provide helpful information. It is likely that if it were ordered, there may be hesitation by the radiologist to perform the scan with contrast because of the patient’s CKD.

Concern for contrast-induced kidney injury has limited diagnostic testing for many years. How strong is the evidence for contrast-induced kidney injury, and should we avoid testing that requires contrast in patients with CKD? McDonald and colleagues performed a meta-analysis with 13 studies meeting inclusion criteria, involving 25,950 patients.1 They found no increased risk of acute kidney injury (AKI) in patients who received contrast medium compared with those who did not receive contrast; relative risk of AKI for those receiving contrast was 0.79 (confidence interval: 0.62-1.02). Importantly, there was no difference in AKI in patients with diabetes or CKD.

Ehmann et al. looked at renal outcomes in patients who received IV contrast when they presented to an emergency department with AKI.2 They found that in patients with AKI, receiving contrast was not associated with persistent AKI at hospital discharge. Hinson and colleagues looked at patients arriving at the emergency department and needing imaging.3 They did a retrospective, cohort analysis of 17,934 patients who had CT with contrast, CT with no contrast, or no CT. Contrast administration was not associated with increased incidence of AKI (odds ratio, 0.96, CI: 0.85-1.08).

Aycock et al. did a meta-analysis of AKI after CT scanning, including 28 studies involving 107,335 patients.4 They found that compared with noncontrast CT, CT scanning with contrast was not associated with AKI (OR, 0.94, CI: 0.83-1.07). Elias and Aronson looked at the risk of AKI after contrast in patients receiving CT scans compared with those who received ventilation/perfusion scans to evaluate for pulmonary embolism.5 There were 44 AKI events (4.5%) in patients exposed to contrast media and 33 events (3.4%) in patients not exposed to contrast media (risk difference: 1.1%, 95% CI: -0.6% to 2.9%; OR, 1.39, CI: 0.86-2.26; P = .18).

Despite multiple studies showing no increased risk, there is still a concern that contrast can cause AKI.6 Animal models have shown iodinated contrast can have a deleterious effect on mitochondria and membrane function.6 Criticisms of the retrospective nature of many of the studies I have shared, and the lack of randomized, controlled trials are that there may be bias in these studies, as the highest-risk patients are the ones most likely not to receive contrast. In a joint guideline from the American College of Radiology and the National Kidney Foundation, this statement was made: “The risk of acute kidney injury developing in patients with reduced kidney function following exposure to intravenous iodinated contrast media has been overstated.”7 Their recommendation was to give contrast if needed in patients with glomerular filtration rates (GFRs) greater than 30.



Myth: Contrast-induced renal injury is a concern.

Clinical impact: For CT scanning, it is OK to give contrast when needed. A conservative cutoff for contrast use would be a GFR less than 30.
 

Dr. Paauw is professor of medicine in the Division of General Internal Medicine at the University of Washington, Seattle, and he serves as third-year medical student clerkship director at the University of Washington. Contact Dr. Paauw at dpaauw@uw.edu.

References

1. McDonald JS et al. Radiology. 2013:267:119-128.

2. Ehmann MR et al. Intensive Care Med. 2023:49(2):205-215.

3. Hinson JS et al. Ann Emerg Med. 2017;69(5):577-586.

4. Aycock RD et al. Ann Emerg Med. 2018 Jan;71(1):44-53.

5. Elias A, Aronson D. Thromb Haemost. 2021 Jun;121(6):800-807.

6. Weisbord SD, du Cheryon D. Intensive Care Med. 2018;44(1):107-109.

7. Davenport MS et al. Radiology. 2020;294(3):660-668.

Publications
Publications
Topics
Article Type
Sections
Teambase XML
<?xml version="1.0" encoding="UTF-8"?>
<!--$RCSfile: InCopy_agile.xsl,v $ $Revision: 1.35 $-->
<!--$RCSfile: drupal.xsl,v $ $Revision: 1.7 $-->
<root generator="drupal.xsl" gversion="1.7"> <header> <fileName>167947</fileName> <TBEID>0C04FEF7.SIG</TBEID> <TBUniqueIdentifier>MD_0C04FEF7</TBUniqueIdentifier> <newsOrJournal>News</newsOrJournal> <publisherName>Frontline Medical Communications</publisherName> <storyname/> <articleType>353</articleType> <TBLocation>QC Done-All Pubs</TBLocation> <QCDate>20240510T181607</QCDate> <firstPublished>20240513T091952</firstPublished> <LastPublished>20240513T091952</LastPublished> <pubStatus qcode="stat:"/> <embargoDate/> <killDate/> <CMSDate>20240513T091952</CMSDate> <articleSource/> <facebookInfo/> <meetingNumber/> <byline>Douglas Paauw</byline> <bylineText>DOUGLAS S. PAAUW, MD</bylineText> <bylineFull>DOUGLAS S. PAAUW, MD</bylineFull> <bylineTitleText/> <USOrGlobal/> <wireDocType/> <newsDocType/> <journalDocType/> <linkLabel/> <pageRange/> <citation/> <quizID/> <indexIssueDate/> <itemClass qcode="ninat:text"/> <provider qcode="provider:imng"> <name>IMNG Medical Media</name> <rightsInfo> <copyrightHolder> <name>Frontline Medical News</name> </copyrightHolder> <copyrightNotice>Copyright (c) 2015 Frontline Medical News, a Frontline Medical Communications Inc. company. All rights reserved. This material may not be published, broadcast, copied, or otherwise reproduced or distributed without the prior written permission of Frontline Medical Communications Inc.</copyrightNotice> </rightsInfo> </provider> <abstract/> <metaDescription>A 59-year-old man presents with abdominal pain. He has a history of small bowel obstruction and diverticulitis. His medical history includes chronic kidney dise</metaDescription> <articlePDF/> <teaserImage>248215</teaserImage> <teaser>For CT scanning, it is OK to give contrast when needed. A conservative cutoff for contrast use would be a GFR less than 30.</teaser> <title>Myth of the Month: Is Contrast-Induced Acute Kidney Injury Real?</title> <deck/> <disclaimer/> <AuthorList/> <articleURL/> <doi/> <pubMedID/> <publishXMLStatus/> <publishXMLVersion>1</publishXMLVersion> <useEISSN>0</useEISSN> <urgency/> <pubPubdateYear/> <pubPubdateMonth/> <pubPubdateDay/> <pubVolume/> <pubNumber/> <wireChannels/> <primaryCMSID/> <CMSIDs/> <keywords/> <seeAlsos/> <publications_g> <publicationData> <publicationCode>mdemed</publicationCode> <pubIssueName/> <pubArticleType/> <pubTopics/> <pubCategories/> <pubSections/> <journalTitle/> <journalFullTitle/> <copyrightStatement/> </publicationData> <publicationData> <publicationCode>fp</publicationCode> <pubIssueName/> <pubArticleType/> <pubTopics/> <pubCategories/> <pubSections/> </publicationData> <publicationData> <publicationCode>im</publicationCode> <pubIssueName/> <pubArticleType/> <pubTopics/> <pubCategories/> <pubSections/> </publicationData> </publications_g> <publications> <term>58877</term> <term>15</term> <term canonical="true">21</term> </publications> <sections> <term>52</term> <term canonical="true">28225</term> </sections> <topics> <term canonical="true">255</term> <term>230</term> </topics> <links> <link> <itemClass qcode="ninat:picture"/> <altRep contenttype="image/jpeg">images/2400c6a0.jpg</altRep> <description role="drol:caption">Dr. Douglas S. Paauw</description> <description role="drol:credit"/> </link> </links> </header> <itemSet> <newsItem> <itemMeta> <itemRole>Main</itemRole> <itemClass>text</itemClass> <title>Myth of the Month: Is Contrast-Induced Acute Kidney Injury Real?</title> <deck/> </itemMeta> <itemContent> <p>A 59-year-old man presents with abdominal pain. He has a history of small bowel obstruction and diverticulitis. His medical history includes chronic kidney disease (CKD; baseline creatinine, 1.8 mg/dL), hypertension, type 2 diabetes, and depression. He had a colectomy 6 years ago for colon cancer.<br/><br/>He takes the following medications: Semaglutide (1 mg weekly), amlodipine (5 mg once daily), and escitalopram (10 mg once daily). On physical exam his blood pressure is 130/80 mm Hg, his pulse is 90, and his temperature is 37.2 degrees C. He has normal bowel sounds but guarding in the right lower quadrant.[[{"fid":"248215","view_mode":"medstat_image_flush_right","fields":{"format":"medstat_image_flush_right","field_file_image_alt_text[und][0][value]":"Dr. Douglas S. Paauw, University of Washington, Seattle","field_file_image_credit[und][0][value]":"","field_file_image_caption[und][0][value]":"Dr. Douglas S. Paauw"},"type":"media","attributes":{"class":"media-element file-medstat_image_flush_right"}}]] <br/><br/>His hemoglobin is 14 g/dL, his blood sodium is 136 mEq/L, his blood potassium is 4.0 mmol/L, his BUN is 26 mg/dL, and his creatinine is 1.9 mg/dL. His kidney, ureter, bladder x-ray is unremarkable.<br/><br/></p> <p>What imaging would you recommend?<br/><br/>A) CT without contrast<br/><br/>B) CT with contrast<br/><br/>C) MRI<br/><br/>D) Abdominal ultrasound</p> <p>This patient has several potential causes for his abdominal pain that imaging may clarify. I think a contrast CT scan would be the most likely to provide helpful information. It is likely that if it were ordered, there may be hesitation by the radiologist to perform the scan with contrast because of the patient’s CKD. <br/><br/>Concern for contrast-induced kidney injury has limited diagnostic testing for many years. How strong is the evidence for contrast-induced kidney injury, and should we avoid testing that requires contrast in patients with CKD? McDonald and colleagues performed a meta-analysis with 13 studies meeting inclusion criteria, involving 25,950 patients.<sup>1 </sup>They found no increased risk of acute kidney injury (AKI) in patients who received contrast medium compared with those who did not receive contrast; relative risk of AKI for those receiving contrast was 0.79 (confidence interval: 0.62-1.02). Importantly, there was no difference in AKI in patients with diabetes or CKD.<br/><br/>Ehmann et al. looked at renal outcomes in patients who received IV contrast when they presented to an emergency department with AKI.<sup>2 </sup>They found that in patients with AKI, receiving contrast was not associated with persistent AKI at hospital discharge. Hinson and colleagues looked at patients arriving at the emergency department and needing imaging.<sup>3</sup> They did a retrospective, cohort analysis of 17,934 patients who had CT with contrast, CT with no contrast, or no CT. Contrast administration was not associated with increased incidence of AKI (odds ratio, 0.96, CI: 0.85-1.08). <br/><br/>Aycock et al. did a meta-analysis of AKI after CT scanning, including 28 studies involving 107,335 patients.<sup>4</sup> They found that compared with noncontrast CT, CT scanning with contrast was not associated with AKI (OR, 0.94, CI: 0.83-1.07). Elias and Aronson looked at the risk of AKI after contrast in patients receiving CT scans compared with those who received ventilation/perfusion scans to evaluate for pulmonary embolism.<sup>5</sup> There were 44 AKI events (4.5%) in patients exposed to contrast media and 33 events (3.4%) in patients not exposed to contrast media (risk difference: 1.1%, 95% CI: -0.6% to 2.9%; OR, 1.39, CI: 0.86-2.26; <em>P</em> = .18).<br/><br/>Despite multiple studies showing no increased risk, there is still a concern that contrast can cause AKI.<sup>6</sup> Animal models have shown iodinated contrast can have a deleterious effect on mitochondria and membrane function.<sup>6</sup> Criticisms of the retrospective nature of many of the studies I have shared, and the lack of randomized, controlled trials are that there may be bias in these studies, as the highest-risk patients are the ones most likely not to receive contrast. In a joint guideline from the American College of Radiology and the National Kidney Foundation, this statement was made: “The risk of acute kidney injury developing in patients with reduced kidney function following exposure to intravenous iodinated contrast media has been overstated.”<sup>7</sup> Their recommendation was to give contrast if needed in patients with glomerular filtration rates (GFRs) greater than 30.<br/><br/><br/><br/>Myth: Contrast-induced renal injury is a concern.<br/><br/>Clinical impact: For CT scanning, it is OK to give contrast when needed. A conservative cutoff for contrast use would be a GFR less than 30.<br/><br/></p> <p> <em>Dr. Paauw is professor of medicine in the Division of General Internal Medicine at the University of Washington, Seattle, and he serves as third-year medical student clerkship director at the University of Washington. Contact Dr. Paauw at <span class="Hyperlink"><a href="mailto:dpaauw%40uw.edu?subject=">dpaauw@uw.edu</a></span>.</em> </p> <h2>References</h2> <p>1. McDonald JS et al. <span class="Hyperlink"><a href="https://pubs.rsna.org/doi/10.1148/radiol.12121460">Radiology. 2013:267:119-128</a></span>.<br/><br/>2. Ehmann MR et al. <span class="Hyperlink"><a href="https://link.springer.com/article/10.1007/s00134-022-06966-w">Intensive Care Med. 2023:49(2):205-215</a></span>.<br/><br/>3. Hinson JS et al. <span class="Hyperlink"><a href="https://www.annemergmed.com/article/S0196-0644(16)31388-9/abstract">Ann Emerg Med. 2017;69(5):577-586</a></span>.<br/><br/>4. Aycock RD et al. <span class="Hyperlink"><a href="https://www.annemergmed.com/article/S0196-0644(17)30881-8/abstract">Ann Emerg Med. 2018 Jan;71(1):44-53</a></span>.<br/><br/>5. Elias A, Aronson D. <span class="Hyperlink"><a href="https://www.thieme-connect.de/products/ejournals/abstract/10.1055/s-0040-1721387">Thromb Haemost. 2021 Jun;121(6):800-807</a></span>.<br/><br/>6. Weisbord SD, du Cheryon D. <span class="Hyperlink"><a href="https://link.springer.com/article/10.1007/s00134-017-5015-6">Intensive Care Med. 2018;44(1):107-109</a></span>.<br/><br/>7. Davenport MS et al. <span class="Hyperlink"><a href="https://pubs.rsna.org/doi/10.1148/radiol.2019192094">Radiology. 2020;294(3):660-668</a></span>.</p> </itemContent> </newsItem> <newsItem> <itemMeta> <itemRole>teaser</itemRole> <itemClass>text</itemClass> <title/> <deck/> </itemMeta> <itemContent> </itemContent> </newsItem> </itemSet></root>
Disallow All Ads
Content Gating
No Gating (article Unlocked/Free)
Alternative CME
Disqus Comments
Default
Use ProPublica
Hide sidebar & use full width
render the right sidebar.
Conference Recap Checkbox
Not Conference Recap
Clinical Edge
Display the Slideshow in this Article
Medscape Article
Display survey writer
Reuters content
Disable Inline Native ads
WebMD Article

Nitroglycerin’s safety and value examined

Article Type
Changed
Mon, 12/19/2022 - 09:29

A 70-year-old man with a history of coronary artery disease (CAD) is seen for concerns about erectile dysfunction (ED). He is requesting sildenafil. He has stable angina, having chest pain with exercise. He uses sublingual nitroglycerin (SL NTG prn) about three times a month. His blood pressure is 140/70 mm Hg. His pulse is 60 beats per minute. His current medications are lisinopril, atorvastatin, aspirin, and SL NTG tablets as needed.

What would you recommend?

A. No sildenafil; refer to urologist for other ED options.

B. Okay to use sildenafil if greater than 6 hours from NTG use.

C. Recommend tadalafil.

Is coprescribing nitrates and phosphodiesterase inhibitors safe?

The FDA warns against the use of phosphodiesterase inhibitors in patients taking nitrates. Combining nitrates with phosphodiesterase type 5 (PDE5) inhibitors is contraindicated because of a synergistic blood pressure lowering effect.1 This warning/contraindication was based on theoretical concerns, as well as concern that of the first 130 deaths reported in patients who took sildenafil, 16 of the patients also were taking nitrates.2

Parker and colleagues studied the safety of giving IV nitroglycerin to patients with coronary artery disease (CAD) who have taken sildenafil.3 The study was a randomized, placebo-controlled, crossover trial. Participants received sildenafil 100 mg or placebo, then received intravenous NTG. Patients who received sildenafil had a 4-6 mm Hg systolic BP drop compared with those who took the placebo. There was no difference in severe events between the sildenafil and placebo groups. The blood levels of nitroglycerin in this study were very likely much higher than the levels that occur with SL NTG.

A recent study by Holt et al. looked at overall cardiovascular outcomes with coprescribing nitrates and phosphodiesterase inhibitors.4 The study was a case crossover design, using a nationwide Danish health registry over the period of 2000-2018. In 2000, the rate of coprescribing of phosphodiesterase inhibitors in ischemic heart disease patients on nitrates was .9 per 100 persons/year and rose to 19.5 prescriptions per 100 persons/year in 2018. During this same time, no statistically significant association was found between the coprescription of nitrates with PDE5 inhibitors and the risk for MI, cardiac arrest, syncope, stroke, or an adverse drug event.
 

Does nitroglycerin response help determine cause of chest pain?

Nitroglycerin response has long been used as a clinical indicator on whether a patient’s chest pain is cardiac or not. Eric A. Shry, MD, and his colleagues looked at the usefulness of nitroglycerin response in the treatment of chest pain as a predictor of ischemic chest pain in an emergency department setting.5

The study was a retrospective review of 223 patients who presented to the emergency department over a 5-month period with ongoing chest pain. They looked at patients who had ongoing chest pain in the emergency department, received nitroglycerin, and did not receive any therapy other than aspirin within 10 minutes of receiving nitroglycerin. Response to the drug was compared with the final diagnosis of cardiac versus noncardiac chest pain.

Of the patients with a final determination of cardiac chest pain, 88% had a nitroglycerin response, whereas 92% of the patients with noncardiac chest pain had a nitroglycerin response (P = .50).

Deborah B. Diercks, MD, and her colleagues looked at improvement in chest pain scores in the emergency department in patients treated with nitroglycerin and whether it correlated with a cardiac etiology of chest pain.6 The study was a prospective, observational study of 664 patients in an urban tertiary care emergency department over a 16-month period. An 11-point numeric chest pain scale was assessed and recorded by research assistants before and 5 minutes after receiving nitroglycerin. The scale ranged from 0 (no pain) to 10 (worst pain imaginable).

A final diagnosis of a cardiac etiology for chest pain was found in 18% of the patients in the study. Of the patients who had cardiac-related chest pain, 20% had no reduction in pain with nitroglycerin, compared with 19% of the patients without cardiac-related chest pain.

A complete or significant reduction in chest pain occurred with nitroglycerin in 31% of patients with cardiac chest pain and 27% of the patients without cardiac chest pain (P = .76).

Nitroglycerin response does not appear to be helpful in distinguishing cardiac from noncardiac chest pain, but a study by His and colleagues offers an interesting twist.7

The authors of this research studied 118 patients looking to see if the side effect of headache with nitroglycerin was more common in patients who did not have CAD than in those who did. All the patients had a varying degree of relief of chest pain with NTG administration within 10 minutes. In patients with normal coronary arteries or minimal CAD, 73% had headache caused by NTG, whereas in patients with obstructive CAD, only 23% had headache after NTG use.
 

 

 

Take-home messages

  • Short acting nitroglycerin may not be a contraindication for phosphodiesterase inhibitor use.
  • More data are still needed.
  • Nitroglycerin response does not help distinguish chest pain from CAD from noncardiac causes.

Dr. Paauw is professor of medicine in the division of general internal medicine at the University of Washington, Seattle, and he serves as third-year medical student clerkship director at the University of Washington. He is a member of the editorial advisory board of Internal Medicine News. Dr. Paauw has no conflicts to disclose. Contact him at imnews@mdedge.com.

References

1. Schwartz BG, Kloner RA. Drug interactions with phosphodiesterase-5 inhibitors used for the treatment of erectile dysfunction or pulmonary hypertension. Circulation. 2010;122:88-95.

2. Kloner RA, Zusman RM. Cardiovascular effects of sildenafil citrate and recommendations for its use. Am J Cardiol. 1999 Sep 9;84(5B):11N-17N.

3. Parker JD et al. Safety of intravenous nitroglycerin after administration of sildenafil citrate to men with coronary artery disease: A double-blind, placebo-controlled, randomized, crossover trial. Crit Care Med. 2007;35:1863-8.

4. Holt A et al. Adverse events associated with coprescription of phosphodiesterase type inhibitors and oral organic nitrates in male patients with ischemic heart disease. Ann Intern Med. 2022 Jun;175(6):774-82.

5. Shry EA et al. Usefulness of the response to sublingual nitroglycerin as a predictor of ischemic chest pain in the emergency department. Am J Cardiol. 2002 Dec 1;90(11):1264-6.

6. Diercks DB et al. Changes in the numeric descriptive scale for pain after sublingual nitroglycerin do not predict cardiac etiology of chest pain. Ann Emerg Med. 2005 Jun;45(6):581-5.

7. His DH et al. Headache response to glyceryl trinitrate in patients with and without obstructive coronary artery disease. Heart 2005;91:1164-6.

Publications
Topics
Sections

A 70-year-old man with a history of coronary artery disease (CAD) is seen for concerns about erectile dysfunction (ED). He is requesting sildenafil. He has stable angina, having chest pain with exercise. He uses sublingual nitroglycerin (SL NTG prn) about three times a month. His blood pressure is 140/70 mm Hg. His pulse is 60 beats per minute. His current medications are lisinopril, atorvastatin, aspirin, and SL NTG tablets as needed.

What would you recommend?

A. No sildenafil; refer to urologist for other ED options.

B. Okay to use sildenafil if greater than 6 hours from NTG use.

C. Recommend tadalafil.

Is coprescribing nitrates and phosphodiesterase inhibitors safe?

The FDA warns against the use of phosphodiesterase inhibitors in patients taking nitrates. Combining nitrates with phosphodiesterase type 5 (PDE5) inhibitors is contraindicated because of a synergistic blood pressure lowering effect.1 This warning/contraindication was based on theoretical concerns, as well as concern that of the first 130 deaths reported in patients who took sildenafil, 16 of the patients also were taking nitrates.2

Parker and colleagues studied the safety of giving IV nitroglycerin to patients with coronary artery disease (CAD) who have taken sildenafil.3 The study was a randomized, placebo-controlled, crossover trial. Participants received sildenafil 100 mg or placebo, then received intravenous NTG. Patients who received sildenafil had a 4-6 mm Hg systolic BP drop compared with those who took the placebo. There was no difference in severe events between the sildenafil and placebo groups. The blood levels of nitroglycerin in this study were very likely much higher than the levels that occur with SL NTG.

A recent study by Holt et al. looked at overall cardiovascular outcomes with coprescribing nitrates and phosphodiesterase inhibitors.4 The study was a case crossover design, using a nationwide Danish health registry over the period of 2000-2018. In 2000, the rate of coprescribing of phosphodiesterase inhibitors in ischemic heart disease patients on nitrates was .9 per 100 persons/year and rose to 19.5 prescriptions per 100 persons/year in 2018. During this same time, no statistically significant association was found between the coprescription of nitrates with PDE5 inhibitors and the risk for MI, cardiac arrest, syncope, stroke, or an adverse drug event.
 

Does nitroglycerin response help determine cause of chest pain?

Nitroglycerin response has long been used as a clinical indicator on whether a patient’s chest pain is cardiac or not. Eric A. Shry, MD, and his colleagues looked at the usefulness of nitroglycerin response in the treatment of chest pain as a predictor of ischemic chest pain in an emergency department setting.5

The study was a retrospective review of 223 patients who presented to the emergency department over a 5-month period with ongoing chest pain. They looked at patients who had ongoing chest pain in the emergency department, received nitroglycerin, and did not receive any therapy other than aspirin within 10 minutes of receiving nitroglycerin. Response to the drug was compared with the final diagnosis of cardiac versus noncardiac chest pain.

Of the patients with a final determination of cardiac chest pain, 88% had a nitroglycerin response, whereas 92% of the patients with noncardiac chest pain had a nitroglycerin response (P = .50).

Deborah B. Diercks, MD, and her colleagues looked at improvement in chest pain scores in the emergency department in patients treated with nitroglycerin and whether it correlated with a cardiac etiology of chest pain.6 The study was a prospective, observational study of 664 patients in an urban tertiary care emergency department over a 16-month period. An 11-point numeric chest pain scale was assessed and recorded by research assistants before and 5 minutes after receiving nitroglycerin. The scale ranged from 0 (no pain) to 10 (worst pain imaginable).

A final diagnosis of a cardiac etiology for chest pain was found in 18% of the patients in the study. Of the patients who had cardiac-related chest pain, 20% had no reduction in pain with nitroglycerin, compared with 19% of the patients without cardiac-related chest pain.

A complete or significant reduction in chest pain occurred with nitroglycerin in 31% of patients with cardiac chest pain and 27% of the patients without cardiac chest pain (P = .76).

Nitroglycerin response does not appear to be helpful in distinguishing cardiac from noncardiac chest pain, but a study by His and colleagues offers an interesting twist.7

The authors of this research studied 118 patients looking to see if the side effect of headache with nitroglycerin was more common in patients who did not have CAD than in those who did. All the patients had a varying degree of relief of chest pain with NTG administration within 10 minutes. In patients with normal coronary arteries or minimal CAD, 73% had headache caused by NTG, whereas in patients with obstructive CAD, only 23% had headache after NTG use.
 

 

 

Take-home messages

  • Short acting nitroglycerin may not be a contraindication for phosphodiesterase inhibitor use.
  • More data are still needed.
  • Nitroglycerin response does not help distinguish chest pain from CAD from noncardiac causes.

Dr. Paauw is professor of medicine in the division of general internal medicine at the University of Washington, Seattle, and he serves as third-year medical student clerkship director at the University of Washington. He is a member of the editorial advisory board of Internal Medicine News. Dr. Paauw has no conflicts to disclose. Contact him at imnews@mdedge.com.

References

1. Schwartz BG, Kloner RA. Drug interactions with phosphodiesterase-5 inhibitors used for the treatment of erectile dysfunction or pulmonary hypertension. Circulation. 2010;122:88-95.

2. Kloner RA, Zusman RM. Cardiovascular effects of sildenafil citrate and recommendations for its use. Am J Cardiol. 1999 Sep 9;84(5B):11N-17N.

3. Parker JD et al. Safety of intravenous nitroglycerin after administration of sildenafil citrate to men with coronary artery disease: A double-blind, placebo-controlled, randomized, crossover trial. Crit Care Med. 2007;35:1863-8.

4. Holt A et al. Adverse events associated with coprescription of phosphodiesterase type inhibitors and oral organic nitrates in male patients with ischemic heart disease. Ann Intern Med. 2022 Jun;175(6):774-82.

5. Shry EA et al. Usefulness of the response to sublingual nitroglycerin as a predictor of ischemic chest pain in the emergency department. Am J Cardiol. 2002 Dec 1;90(11):1264-6.

6. Diercks DB et al. Changes in the numeric descriptive scale for pain after sublingual nitroglycerin do not predict cardiac etiology of chest pain. Ann Emerg Med. 2005 Jun;45(6):581-5.

7. His DH et al. Headache response to glyceryl trinitrate in patients with and without obstructive coronary artery disease. Heart 2005;91:1164-6.

A 70-year-old man with a history of coronary artery disease (CAD) is seen for concerns about erectile dysfunction (ED). He is requesting sildenafil. He has stable angina, having chest pain with exercise. He uses sublingual nitroglycerin (SL NTG prn) about three times a month. His blood pressure is 140/70 mm Hg. His pulse is 60 beats per minute. His current medications are lisinopril, atorvastatin, aspirin, and SL NTG tablets as needed.

What would you recommend?

A. No sildenafil; refer to urologist for other ED options.

B. Okay to use sildenafil if greater than 6 hours from NTG use.

C. Recommend tadalafil.

Is coprescribing nitrates and phosphodiesterase inhibitors safe?

The FDA warns against the use of phosphodiesterase inhibitors in patients taking nitrates. Combining nitrates with phosphodiesterase type 5 (PDE5) inhibitors is contraindicated because of a synergistic blood pressure lowering effect.1 This warning/contraindication was based on theoretical concerns, as well as concern that of the first 130 deaths reported in patients who took sildenafil, 16 of the patients also were taking nitrates.2

Parker and colleagues studied the safety of giving IV nitroglycerin to patients with coronary artery disease (CAD) who have taken sildenafil.3 The study was a randomized, placebo-controlled, crossover trial. Participants received sildenafil 100 mg or placebo, then received intravenous NTG. Patients who received sildenafil had a 4-6 mm Hg systolic BP drop compared with those who took the placebo. There was no difference in severe events between the sildenafil and placebo groups. The blood levels of nitroglycerin in this study were very likely much higher than the levels that occur with SL NTG.

A recent study by Holt et al. looked at overall cardiovascular outcomes with coprescribing nitrates and phosphodiesterase inhibitors.4 The study was a case crossover design, using a nationwide Danish health registry over the period of 2000-2018. In 2000, the rate of coprescribing of phosphodiesterase inhibitors in ischemic heart disease patients on nitrates was .9 per 100 persons/year and rose to 19.5 prescriptions per 100 persons/year in 2018. During this same time, no statistically significant association was found between the coprescription of nitrates with PDE5 inhibitors and the risk for MI, cardiac arrest, syncope, stroke, or an adverse drug event.
 

Does nitroglycerin response help determine cause of chest pain?

Nitroglycerin response has long been used as a clinical indicator on whether a patient’s chest pain is cardiac or not. Eric A. Shry, MD, and his colleagues looked at the usefulness of nitroglycerin response in the treatment of chest pain as a predictor of ischemic chest pain in an emergency department setting.5

The study was a retrospective review of 223 patients who presented to the emergency department over a 5-month period with ongoing chest pain. They looked at patients who had ongoing chest pain in the emergency department, received nitroglycerin, and did not receive any therapy other than aspirin within 10 minutes of receiving nitroglycerin. Response to the drug was compared with the final diagnosis of cardiac versus noncardiac chest pain.

Of the patients with a final determination of cardiac chest pain, 88% had a nitroglycerin response, whereas 92% of the patients with noncardiac chest pain had a nitroglycerin response (P = .50).

Deborah B. Diercks, MD, and her colleagues looked at improvement in chest pain scores in the emergency department in patients treated with nitroglycerin and whether it correlated with a cardiac etiology of chest pain.6 The study was a prospective, observational study of 664 patients in an urban tertiary care emergency department over a 16-month period. An 11-point numeric chest pain scale was assessed and recorded by research assistants before and 5 minutes after receiving nitroglycerin. The scale ranged from 0 (no pain) to 10 (worst pain imaginable).

A final diagnosis of a cardiac etiology for chest pain was found in 18% of the patients in the study. Of the patients who had cardiac-related chest pain, 20% had no reduction in pain with nitroglycerin, compared with 19% of the patients without cardiac-related chest pain.

A complete or significant reduction in chest pain occurred with nitroglycerin in 31% of patients with cardiac chest pain and 27% of the patients without cardiac chest pain (P = .76).

Nitroglycerin response does not appear to be helpful in distinguishing cardiac from noncardiac chest pain, but a study by His and colleagues offers an interesting twist.7

The authors of this research studied 118 patients looking to see if the side effect of headache with nitroglycerin was more common in patients who did not have CAD than in those who did. All the patients had a varying degree of relief of chest pain with NTG administration within 10 minutes. In patients with normal coronary arteries or minimal CAD, 73% had headache caused by NTG, whereas in patients with obstructive CAD, only 23% had headache after NTG use.
 

 

 

Take-home messages

  • Short acting nitroglycerin may not be a contraindication for phosphodiesterase inhibitor use.
  • More data are still needed.
  • Nitroglycerin response does not help distinguish chest pain from CAD from noncardiac causes.

Dr. Paauw is professor of medicine in the division of general internal medicine at the University of Washington, Seattle, and he serves as third-year medical student clerkship director at the University of Washington. He is a member of the editorial advisory board of Internal Medicine News. Dr. Paauw has no conflicts to disclose. Contact him at imnews@mdedge.com.

References

1. Schwartz BG, Kloner RA. Drug interactions with phosphodiesterase-5 inhibitors used for the treatment of erectile dysfunction or pulmonary hypertension. Circulation. 2010;122:88-95.

2. Kloner RA, Zusman RM. Cardiovascular effects of sildenafil citrate and recommendations for its use. Am J Cardiol. 1999 Sep 9;84(5B):11N-17N.

3. Parker JD et al. Safety of intravenous nitroglycerin after administration of sildenafil citrate to men with coronary artery disease: A double-blind, placebo-controlled, randomized, crossover trial. Crit Care Med. 2007;35:1863-8.

4. Holt A et al. Adverse events associated with coprescription of phosphodiesterase type inhibitors and oral organic nitrates in male patients with ischemic heart disease. Ann Intern Med. 2022 Jun;175(6):774-82.

5. Shry EA et al. Usefulness of the response to sublingual nitroglycerin as a predictor of ischemic chest pain in the emergency department. Am J Cardiol. 2002 Dec 1;90(11):1264-6.

6. Diercks DB et al. Changes in the numeric descriptive scale for pain after sublingual nitroglycerin do not predict cardiac etiology of chest pain. Ann Emerg Med. 2005 Jun;45(6):581-5.

7. His DH et al. Headache response to glyceryl trinitrate in patients with and without obstructive coronary artery disease. Heart 2005;91:1164-6.

Publications
Publications
Topics
Article Type
Sections
Teambase XML
<?xml version="1.0" encoding="UTF-8"?>
<!--$RCSfile: InCopy_agile.xsl,v $ $Revision: 1.35 $-->
<!--$RCSfile: drupal.xsl,v $ $Revision: 1.7 $-->
<root generator="drupal.xsl" gversion="1.7"> <header> <fileName>161496</fileName> <TBEID>0C0474A3.SIG</TBEID> <TBUniqueIdentifier>MD_0C0474A3</TBUniqueIdentifier> <newsOrJournal>News</newsOrJournal> <publisherName>Frontline Medical Communications</publisherName> <storyname/> <articleType>2</articleType> <TBLocation>QC Done-All Pubs</TBLocation> <QCDate>20221215T114441</QCDate> <firstPublished>20221215T131350</firstPublished> <LastPublished>20221215T131350</LastPublished> <pubStatus qcode="stat:"/> <embargoDate/> <killDate/> <CMSDate>20221215T131350</CMSDate> <articleSource/> <facebookInfo/> <meetingNumber/> <byline/> <bylineText>DOUGLAS S. PAAUW, MD</bylineText> <bylineFull>DOUGLAS S. PAAUW, MD</bylineFull> <bylineTitleText>MDedge News</bylineTitleText> <USOrGlobal/> <wireDocType/> <newsDocType>Opinion</newsDocType> <journalDocType/> <linkLabel/> <pageRange/> <citation/> <quizID/> <indexIssueDate/> <itemClass qcode="ninat:text"/> <provider qcode="provider:imng"> <name>IMNG Medical Media</name> <rightsInfo> <copyrightHolder> <name>Frontline Medical News</name> </copyrightHolder> <copyrightNotice>Copyright (c) 2015 Frontline Medical News, a Frontline Medical Communications Inc. company. All rights reserved. This material may not be published, broadcast, copied, or otherwise reproduced or distributed without the prior written permission of Frontline Medical Communications Inc.</copyrightNotice> </rightsInfo> </provider> <abstract/> <metaDescription>A 70-year-old man with a history of coronary artery disease (CAD) is seen for concerns about erectile dysfunction (ED). He is requesting sildenafil.</metaDescription> <articlePDF/> <teaserImage/> <teaser>Is it safe for a man with coronary artery disease who takes nitroglycerin to also take sildenafil?</teaser> <title>Nitroglycerin’s safety and value examined</title> <deck/> <disclaimer/> <AuthorList/> <articleURL/> <doi/> <pubMedID/> <publishXMLStatus/> <publishXMLVersion>1</publishXMLVersion> <useEISSN>0</useEISSN> <urgency/> <pubPubdateYear/> <pubPubdateMonth/> <pubPubdateDay/> <pubVolume/> <pubNumber/> <wireChannels/> <primaryCMSID/> <CMSIDs/> <keywords/> <seeAlsos/> <publications_g> <publicationData> <publicationCode>im</publicationCode> <pubIssueName/> <pubArticleType/> <pubTopics/> <pubCategories/> <pubSections/> </publicationData> <publicationData> <publicationCode>fp</publicationCode> <pubIssueName/> <pubArticleType/> <pubTopics/> <pubCategories/> <pubSections/> </publicationData> <publicationData> <publicationCode>card</publicationCode> <pubIssueName/> <pubArticleType/> <pubTopics/> <pubCategories/> <pubSections/> </publicationData> </publications_g> <publications> <term canonical="true">21</term> <term>15</term> <term>5</term> </publications> <sections> <term canonical="true">28225</term> <term>52</term> <term>41022</term> </sections> <topics> <term canonical="true">246</term> <term>194</term> <term>268</term> <term>193</term> </topics> <links/> </header> <itemSet> <newsItem> <itemMeta> <itemRole>Main</itemRole> <itemClass>text</itemClass> <title>Nitroglycerin’s safety and value examined</title> <deck/> </itemMeta> <itemContent> <p><span class="tag metaDescription">A 70-year-old man with a history of coronary artery disease (CAD) is seen for concerns about erectile dysfunction (ED). He is requesting sildenafil.</span> He has stable angina, having chest pain with exercise. He uses sublingual nitroglycerin (SL NTG prn) about three times a month. His blood pressure is 140/70 mm Hg. His pulse is 60 beats per minute. His current medications are lisinopril, atorvastatin, aspirin, and SL NTG tablets as needed.</p> <h2>What would you recommend?</h2> <p>A. No sildenafil; refer to urologist for other ED options.<br/><br/>B. Okay to use sildenafil if greater than 6 hours from NTG use.<br/><br/>C. Recommend tadalafil.</p> <h2>Is coprescribing nitrates and phosphodiesterase inhibitors safe?</h2> <p>The FDA warns against the use of phosphodiesterase inhibitors in patients taking nitrates. Combining nitrates with phosphodiesterase type 5 (PDE5) inhibitors is contraindicated because of a synergistic blood pressure lowering effect.<sup>1</sup> This warning/contraindication was based on theoretical concerns, as well as concern that of the first 130 deaths reported in patients who took sildenafil, 16 of the patients also were taking nitrates.<sup>2</sup> </p> <p>Parker and colleagues studied the safety of giving IV nitroglycerin to patients with coronary artery disease (CAD) who have taken sildenafil.<sup>3</sup> The study was a randomized, placebo-controlled, crossover trial. Participants received sildenafil 100 mg or placebo, then received intravenous NTG. Patients who received sildenafil had a 4-6 mm Hg systolic BP drop compared with those who took the placebo. There was no difference in severe events between the sildenafil and placebo groups. The blood levels of nitroglycerin in this study were very likely much higher than the levels that occur with SL NTG.<br/><br/>A recent study by Holt et al. looked at overall cardiovascular outcomes with coprescribing nitrates and phosphodiesterase inhibitors.<sup>4</sup> The study was a case crossover design, using a nationwide Danish health registry over the period of 2000-2018. In 2000, the rate of coprescribing of phosphodiesterase inhibitors in ischemic heart disease patients on nitrates was .9 per 100 persons/year and rose to 19.5 prescriptions per 100 persons/year in 2018. During this same time, no statistically significant association was found between the coprescription of nitrates with PDE5 inhibitors and the risk for MI, cardiac arrest, syncope, stroke, or an adverse drug event.<br/><br/></p> <h2>Does nitroglycerin response help determine cause of chest pain?</h2> <p>Nitroglycerin response has long been used as a clinical indicator on whether a patient’s chest pain is cardiac or not. Eric A. Shry, MD, and his colleagues looked at the usefulness of nitroglycerin response in the treatment of chest pain as a predictor of ischemic chest pain in an emergency department setting.<sup>5</sup></p> <p>The study was a retrospective review of 223 patients who presented to the emergency department over a 5-month period with ongoing chest pain. They looked at patients who had ongoing chest pain in the emergency department, received nitroglycerin, and did not receive any therapy other than aspirin within 10 minutes of receiving nitroglycerin. Response to the drug was compared with the final diagnosis of cardiac versus noncardiac chest pain.<br/><br/>Of the patients with a final determination of cardiac chest pain, 88% had a nitroglycerin response, whereas 92% of the patients with noncardiac chest pain had a nitroglycerin response (<em>P</em> = .50).<br/><br/>Deborah B. Diercks, MD, and her colleagues looked at improvement in chest pain scores in the emergency department in patients treated with nitroglycerin and whether it correlated with a cardiac etiology of chest pain.<sup>6</sup> The study was a prospective, observational study of 664 patients in an urban tertiary care emergency department over a 16-month period. An 11-point numeric chest pain scale was assessed and recorded by research assistants before and 5 minutes after receiving nitroglycerin. The scale ranged from 0 (no pain) to 10 (worst pain imaginable).<br/><br/>A final diagnosis of a cardiac etiology for chest pain was found in 18% of the patients in the study. Of the patients who had cardiac-related chest pain, 20% had no reduction in pain with nitroglycerin, compared with 19% of the patients without cardiac-related chest pain.<br/><br/>A complete or significant reduction in chest pain occurred with nitroglycerin in 31% of patients with cardiac chest pain and 27% of the patients without cardiac chest pain (<em>P</em> = .76). <br/><br/>Nitroglycerin response does not appear to be helpful in distinguishing cardiac from noncardiac chest pain, but a study by His and colleagues offers an interesting twist.<sup>7</sup> <br/><br/>The authors of this research studied 118 patients looking to see if the side effect of headache with nitroglycerin was more common in patients who did not have CAD than in those who did. All the patients had a varying degree of relief of chest pain with NTG administration within 10 minutes. In patients with normal coronary arteries or minimal CAD, 73% had headache caused by NTG, whereas in patients with obstructive CAD, only 23% had headache after NTG use.<br/><br/></p> <h2>Take-home messages</h2> <ul class="body"> <li>Short acting nitroglycerin may not be a contraindication for phosphodiesterase inhibitor use.</li> <li>More data are still needed.</li> <li>Nitroglycerin response does not help distinguish chest pain from CAD from noncardiac causes.</li> </ul> <p> <em>Dr. Paauw is professor of medicine in the division of general internal medicine at the University of Washington, Seattle, and he serves as third-year medical student clerkship director at the University of Washington. He is a member of the editorial advisory board of Internal Medicine News. Dr. Paauw has no conflicts to disclose. Contact him at imnews@mdedge.com.</em> </p> <h2>References</h2> <p>1. Schwartz BG, Kloner RA. Drug interactions with phosphodiesterase-5 inhibitors used for the treatment of erectile dysfunction or pulmonary hypertension. Circulation. 2010;122:88-95.<br/><br/>2. Kloner RA, Zusman RM. Cardiovascular effects of sildenafil citrate and recommendations for its use. Am J Cardiol. 1999 Sep 9;84(5B):11N-17N.<br/><br/>3. Parker JD et al. Safety of intravenous nitroglycerin after administration of sildenafil citrate to men with coronary artery disease: A double-blind, placebo-controlled, randomized, crossover trial. Crit Care Med. 2007;35:1863-8.<br/><br/>4. Holt A et al. Adverse events associated with coprescription of phosphodiesterase type inhibitors and oral organic nitrates in male patients with ischemic heart disease. Ann Intern Med. 2022 Jun;175(6):774-82.<br/><br/>5. Shry EA et al. Usefulness of the response to sublingual nitroglycerin as a predictor of ischemic chest pain in the emergency department. Am J Cardiol. 2002 Dec 1;90(11):1264-6.<br/><br/>6. Diercks DB et al. Changes in the numeric descriptive scale for pain after sublingual nitroglycerin do not predict cardiac etiology of chest pain. Ann Emerg Med. 2005 Jun;45(6):581-5.<br/><br/>7. His DH et al. Headache response to glyceryl trinitrate in patients with and without obstructive coronary artery disease. Heart 2005;91:1164-6.</p> </itemContent> </newsItem> <newsItem> <itemMeta> <itemRole>teaser</itemRole> <itemClass>text</itemClass> <title/> <deck/> </itemMeta> <itemContent> </itemContent> </newsItem> </itemSet></root>
Disallow All Ads
Content Gating
No Gating (article Unlocked/Free)
Alternative CME
Disqus Comments
Default
Use ProPublica
Hide sidebar & use full width
render the right sidebar.
Conference Recap Checkbox
Not Conference Recap
Clinical Edge
Display the Slideshow in this Article
Medscape Article
Display survey writer
Reuters content
Disable Inline Native ads
WebMD Article

Do lipid labs need to be fasting?

Article Type
Changed
Fri, 09/30/2022 - 13:46

When I worked as a scribe prior to starting medical school, it was commonplace for patients to have fasting labs. I always felt terrible for the patients we saw late in the afternoon that had somehow fasted all day. For many other patients, there was the challenge of finding a time when they could return to have fasting labs drawn.

While in medical school, I have seen the transition of my preceptors’ recommendations, where it seems patients can now have nonfasting labs. However, I have still observed instances when patients need to have fasting labs. We can look at an example case to better understand when and why patients do and do not need to fast prior to having their lipids checked.

Ervin_Kendall_SEATTLE_web.jpg
Kendall Ervin

 

Case

A 57-year-old woman presents for an annual wellness visit. She has been healthy this past year with no new concerns. Her blood pressure has been well controlled, and she continues on a statin for hyperlipidemia. She is due for annual labs. She ate breakfast this morning. Which of the following do you recommend?

A. Obtain lipids with her other blood work now.

B. Have her return tomorrow to obtain fasting labs.

In this situation, A is the correct answer. The patient is due for routine screening labs and there are no current indications that fasting labs are necessary.

Studies of fasting vs. nonfasting lipids

Sidhu and Naugler performed a cross-sectional analysis comparing lipid values at fasting intervals of 1 hour to 16 hours.1 They found the mean total cholesterol and HDL cholesterol values differed by greater than 2%. For LDL cholesterol, the values differed by less than 10% and triglycerides values differed by less than 20%. With this information, the researchers concluded fasting for routine lipids is generally unnecessary.

Mora and colleagues performed a post hoc prospective follow-up of a randomized control

Paauw_Doug_SEATTLE_2019_web2.jpg
Dr. Douglas S. Paauw

trial to assess if nonfasting lipid measurements could cause misclassification of cardiovascular risk assessment.2 Based on 8,270 participants, coronary events associated with fasting vs. nonfasting lipid values were similar when adjusted hazard ratios were compared. They also found an agreement of 94.8% when classifying participants into ASCVD risk categories for fasting and nonfasting lipid values. These outcomes led them to support the use of nonfasting lipid labs for routine cardiovascular risk assessment.

Rahman and colleagues performed a systematic review and found the use of nonfasting lipid values can reliably determine statin management in most situations.3 Circumstances where fasting labs should be used are if patients have a genetic dyslipidemia, if patients have severe hypertriglyceridemia (greater than 500 mg/dL), and if patients have pancreatitis. Triglyceride values fluctuate the most between the fasting and nonfasting state as seen above from Sidhu and Naugler. This could impact triglyceride disorder management and the accuracy of LDL cholesterol estimation (calculated by the Friedewald equation: LDL cholesterol = total cholesterol – HDL cholesterol – triglycerides/5 in mg/dL).3

 

 

Benefits of nonfasting lipid labs

There are many benefits of nonfasting labs. For the patients, they do not have to come to their appointments hungry, we can reduce the risk of hypoglycemia for those with diabetes, and they do not have to come back at a later date if they ate something earlier in the day.

For the lab, we can improve efficiency and decrease early morning congestion when patients typically come in for fasting labs.

Lastly, for the provider, nonfasting labs can improve workflow and help decrease the number of patients lost to follow-up who were unable to complete fasting labs the same day as their appointment.
 

Summary

Patients do not need to fast prior to having lipid levels drawn for routine screening. Fasting labs should be considered for patients who have a genetic dyslipidemia or if there is concern for hypertriglyceridemia.

Per the ACC/AHA guidelines, nonfasting lipids can be used to assess ASCVD risk and to establish a baseline LDL cholesterol in adults 20 years and older. If a patient has nonfasting triglycerides greater than 400 mg/dL, repeat fasting lipids should be drawn to assess fasting triglycerides and to establish a baseline LDL cholesterol.4
 

Ms. Ervin is a fourth-year medical student at the University of Washington, Seattle. She has no conflicts to disclose. Dr. Paauw is professor of medicine in the division of general internal medicine at the University of Washington, and he serves as third-year medical student clerkship director at the university. He is a member of the editorial advisory board of Internal Medicine News. Dr. Paauw has no conflicts to disclose. Contact him at imnews@mdedge.com.

References

1. Rahman F et al. Curr Atheroscler Rep. 2018;20(3):14. Published 2018 Feb 17.

2. Mora S et al. JAMA Intern Med. 2019;179(7):898-905.

3. Sidhu D and Naugler C. Arch Intern Med. 2012;172(22):1707-10.

4. Hoover LE. Am Fam Physician. 2019 May 1;99(9):589-91.

Publications
Topics
Sections

When I worked as a scribe prior to starting medical school, it was commonplace for patients to have fasting labs. I always felt terrible for the patients we saw late in the afternoon that had somehow fasted all day. For many other patients, there was the challenge of finding a time when they could return to have fasting labs drawn.

While in medical school, I have seen the transition of my preceptors’ recommendations, where it seems patients can now have nonfasting labs. However, I have still observed instances when patients need to have fasting labs. We can look at an example case to better understand when and why patients do and do not need to fast prior to having their lipids checked.

Ervin_Kendall_SEATTLE_web.jpg
Kendall Ervin

 

Case

A 57-year-old woman presents for an annual wellness visit. She has been healthy this past year with no new concerns. Her blood pressure has been well controlled, and she continues on a statin for hyperlipidemia. She is due for annual labs. She ate breakfast this morning. Which of the following do you recommend?

A. Obtain lipids with her other blood work now.

B. Have her return tomorrow to obtain fasting labs.

In this situation, A is the correct answer. The patient is due for routine screening labs and there are no current indications that fasting labs are necessary.

Studies of fasting vs. nonfasting lipids

Sidhu and Naugler performed a cross-sectional analysis comparing lipid values at fasting intervals of 1 hour to 16 hours.1 They found the mean total cholesterol and HDL cholesterol values differed by greater than 2%. For LDL cholesterol, the values differed by less than 10% and triglycerides values differed by less than 20%. With this information, the researchers concluded fasting for routine lipids is generally unnecessary.

Mora and colleagues performed a post hoc prospective follow-up of a randomized control

Paauw_Doug_SEATTLE_2019_web2.jpg
Dr. Douglas S. Paauw

trial to assess if nonfasting lipid measurements could cause misclassification of cardiovascular risk assessment.2 Based on 8,270 participants, coronary events associated with fasting vs. nonfasting lipid values were similar when adjusted hazard ratios were compared. They also found an agreement of 94.8% when classifying participants into ASCVD risk categories for fasting and nonfasting lipid values. These outcomes led them to support the use of nonfasting lipid labs for routine cardiovascular risk assessment.

Rahman and colleagues performed a systematic review and found the use of nonfasting lipid values can reliably determine statin management in most situations.3 Circumstances where fasting labs should be used are if patients have a genetic dyslipidemia, if patients have severe hypertriglyceridemia (greater than 500 mg/dL), and if patients have pancreatitis. Triglyceride values fluctuate the most between the fasting and nonfasting state as seen above from Sidhu and Naugler. This could impact triglyceride disorder management and the accuracy of LDL cholesterol estimation (calculated by the Friedewald equation: LDL cholesterol = total cholesterol – HDL cholesterol – triglycerides/5 in mg/dL).3

 

 

Benefits of nonfasting lipid labs

There are many benefits of nonfasting labs. For the patients, they do not have to come to their appointments hungry, we can reduce the risk of hypoglycemia for those with diabetes, and they do not have to come back at a later date if they ate something earlier in the day.

For the lab, we can improve efficiency and decrease early morning congestion when patients typically come in for fasting labs.

Lastly, for the provider, nonfasting labs can improve workflow and help decrease the number of patients lost to follow-up who were unable to complete fasting labs the same day as their appointment.
 

Summary

Patients do not need to fast prior to having lipid levels drawn for routine screening. Fasting labs should be considered for patients who have a genetic dyslipidemia or if there is concern for hypertriglyceridemia.

Per the ACC/AHA guidelines, nonfasting lipids can be used to assess ASCVD risk and to establish a baseline LDL cholesterol in adults 20 years and older. If a patient has nonfasting triglycerides greater than 400 mg/dL, repeat fasting lipids should be drawn to assess fasting triglycerides and to establish a baseline LDL cholesterol.4
 

Ms. Ervin is a fourth-year medical student at the University of Washington, Seattle. She has no conflicts to disclose. Dr. Paauw is professor of medicine in the division of general internal medicine at the University of Washington, and he serves as third-year medical student clerkship director at the university. He is a member of the editorial advisory board of Internal Medicine News. Dr. Paauw has no conflicts to disclose. Contact him at imnews@mdedge.com.

References

1. Rahman F et al. Curr Atheroscler Rep. 2018;20(3):14. Published 2018 Feb 17.

2. Mora S et al. JAMA Intern Med. 2019;179(7):898-905.

3. Sidhu D and Naugler C. Arch Intern Med. 2012;172(22):1707-10.

4. Hoover LE. Am Fam Physician. 2019 May 1;99(9):589-91.

When I worked as a scribe prior to starting medical school, it was commonplace for patients to have fasting labs. I always felt terrible for the patients we saw late in the afternoon that had somehow fasted all day. For many other patients, there was the challenge of finding a time when they could return to have fasting labs drawn.

While in medical school, I have seen the transition of my preceptors’ recommendations, where it seems patients can now have nonfasting labs. However, I have still observed instances when patients need to have fasting labs. We can look at an example case to better understand when and why patients do and do not need to fast prior to having their lipids checked.

Ervin_Kendall_SEATTLE_web.jpg
Kendall Ervin

 

Case

A 57-year-old woman presents for an annual wellness visit. She has been healthy this past year with no new concerns. Her blood pressure has been well controlled, and she continues on a statin for hyperlipidemia. She is due for annual labs. She ate breakfast this morning. Which of the following do you recommend?

A. Obtain lipids with her other blood work now.

B. Have her return tomorrow to obtain fasting labs.

In this situation, A is the correct answer. The patient is due for routine screening labs and there are no current indications that fasting labs are necessary.

Studies of fasting vs. nonfasting lipids

Sidhu and Naugler performed a cross-sectional analysis comparing lipid values at fasting intervals of 1 hour to 16 hours.1 They found the mean total cholesterol and HDL cholesterol values differed by greater than 2%. For LDL cholesterol, the values differed by less than 10% and triglycerides values differed by less than 20%. With this information, the researchers concluded fasting for routine lipids is generally unnecessary.

Mora and colleagues performed a post hoc prospective follow-up of a randomized control

Paauw_Doug_SEATTLE_2019_web2.jpg
Dr. Douglas S. Paauw

trial to assess if nonfasting lipid measurements could cause misclassification of cardiovascular risk assessment.2 Based on 8,270 participants, coronary events associated with fasting vs. nonfasting lipid values were similar when adjusted hazard ratios were compared. They also found an agreement of 94.8% when classifying participants into ASCVD risk categories for fasting and nonfasting lipid values. These outcomes led them to support the use of nonfasting lipid labs for routine cardiovascular risk assessment.

Rahman and colleagues performed a systematic review and found the use of nonfasting lipid values can reliably determine statin management in most situations.3 Circumstances where fasting labs should be used are if patients have a genetic dyslipidemia, if patients have severe hypertriglyceridemia (greater than 500 mg/dL), and if patients have pancreatitis. Triglyceride values fluctuate the most between the fasting and nonfasting state as seen above from Sidhu and Naugler. This could impact triglyceride disorder management and the accuracy of LDL cholesterol estimation (calculated by the Friedewald equation: LDL cholesterol = total cholesterol – HDL cholesterol – triglycerides/5 in mg/dL).3

 

 

Benefits of nonfasting lipid labs

There are many benefits of nonfasting labs. For the patients, they do not have to come to their appointments hungry, we can reduce the risk of hypoglycemia for those with diabetes, and they do not have to come back at a later date if they ate something earlier in the day.

For the lab, we can improve efficiency and decrease early morning congestion when patients typically come in for fasting labs.

Lastly, for the provider, nonfasting labs can improve workflow and help decrease the number of patients lost to follow-up who were unable to complete fasting labs the same day as their appointment.
 

Summary

Patients do not need to fast prior to having lipid levels drawn for routine screening. Fasting labs should be considered for patients who have a genetic dyslipidemia or if there is concern for hypertriglyceridemia.

Per the ACC/AHA guidelines, nonfasting lipids can be used to assess ASCVD risk and to establish a baseline LDL cholesterol in adults 20 years and older. If a patient has nonfasting triglycerides greater than 400 mg/dL, repeat fasting lipids should be drawn to assess fasting triglycerides and to establish a baseline LDL cholesterol.4
 

Ms. Ervin is a fourth-year medical student at the University of Washington, Seattle. She has no conflicts to disclose. Dr. Paauw is professor of medicine in the division of general internal medicine at the University of Washington, and he serves as third-year medical student clerkship director at the university. He is a member of the editorial advisory board of Internal Medicine News. Dr. Paauw has no conflicts to disclose. Contact him at imnews@mdedge.com.

References

1. Rahman F et al. Curr Atheroscler Rep. 2018;20(3):14. Published 2018 Feb 17.

2. Mora S et al. JAMA Intern Med. 2019;179(7):898-905.

3. Sidhu D and Naugler C. Arch Intern Med. 2012;172(22):1707-10.

4. Hoover LE. Am Fam Physician. 2019 May 1;99(9):589-91.

Publications
Publications
Topics
Article Type
Sections
Teambase XML
<?xml version="1.0" encoding="UTF-8"?>
<!--$RCSfile: InCopy_agile.xsl,v $ $Revision: 1.35 $-->
<!--$RCSfile: drupal.xsl,v $ $Revision: 1.7 $-->
<root generator="drupal.xsl" gversion="1.7"> <header> <fileName>160058</fileName> <TBEID>0C0454EB.SIG</TBEID> <TBUniqueIdentifier>MD_0C0454EB</TBUniqueIdentifier> <newsOrJournal>News</newsOrJournal> <publisherName>Frontline Medical Communications</publisherName> <storyname>Do lipid labs need to be fasting</storyname> <articleType>2</articleType> <TBLocation>QC Done-All Pubs</TBLocation> <QCDate>20220921T093119</QCDate> <firstPublished>20220921T110226</firstPublished> <LastPublished>20220921T110226</LastPublished> <pubStatus qcode="stat:"/> <embargoDate/> <killDate/> <CMSDate>20220921T110226</CMSDate> <articleSource/> <facebookInfo/> <meetingNumber/> <byline>Ervin and Paauw</byline> <bylineText>KENDALL ERVIN AND DOUGLAS S. PAAUW, MD</bylineText> <bylineFull>KENDALL ERVIN AND DOUGLAS S. PAAUW, MD</bylineFull> <bylineTitleText/> <USOrGlobal/> <wireDocType/> <newsDocType>Column</newsDocType> <journalDocType/> <linkLabel/> <pageRange/> <citation/> <quizID/> <indexIssueDate/> <itemClass qcode="ninat:text"/> <provider qcode="provider:imng"> <name>IMNG Medical Media</name> <rightsInfo> <copyrightHolder> <name>Frontline Medical News</name> </copyrightHolder> <copyrightNotice>Copyright (c) 2015 Frontline Medical News, a Frontline Medical Communications Inc. company. All rights reserved. This material may not be published, broadcast, copied, or otherwise reproduced or distributed without the prior written permission of Frontline Medical Communications Inc.</copyrightNotice> </rightsInfo> </provider> <abstract/> <metaDescription>While in medical school, I have seen the transition of my preceptors’ recommendations, where it seems patients can now have nonfasting labs.</metaDescription> <articlePDF/> <teaserImage>289532</teaserImage> <teaser>There are instances when patients need to have fasting labs.</teaser> <title>Do lipid labs need to be fasting?</title> <deck/> <disclaimer/> <AuthorList/> <articleURL/> <doi/> <pubMedID/> <publishXMLStatus/> <publishXMLVersion>1</publishXMLVersion> <useEISSN>0</useEISSN> <urgency/> <pubPubdateYear/> <pubPubdateMonth/> <pubPubdateDay/> <pubVolume/> <pubNumber/> <wireChannels/> <primaryCMSID/> <CMSIDs/> <keywords/> <seeAlsos/> <publications_g> <publicationData> <publicationCode>im</publicationCode> <pubIssueName/> <pubArticleType/> <pubTopics/> <pubCategories/> <pubSections/> </publicationData> <publicationData> <publicationCode>fp</publicationCode> <pubIssueName/> <pubArticleType/> <pubTopics/> <pubCategories/> <pubSections/> </publicationData> <publicationData> <publicationCode>card</publicationCode> <pubIssueName/> <pubArticleType/> <pubTopics/> <pubCategories/> <pubSections/> </publicationData> </publications_g> <publications> <term canonical="true">21</term> <term>15</term> <term>5</term> </publications> <sections> <term canonical="true">28225</term> <term>52</term> <term>41022</term> </sections> <topics> <term canonical="true">280</term> <term>194</term> <term>239</term> </topics> <links> <link> <itemClass qcode="ninat:picture"/> <altRep contenttype="image/jpeg">images/240112c7.jpg</altRep> <description role="drol:caption">Kendall Ervin</description> <description role="drol:credit"/> </link> <link> <itemClass qcode="ninat:picture"/> <altRep contenttype="image/jpeg">images/2400c6be.jpg</altRep> <description role="drol:caption">Dr. Douglas S. Paauw</description> <description role="drol:credit"/> </link> </links> </header> <itemSet> <newsItem> <itemMeta> <itemRole>Main</itemRole> <itemClass>text</itemClass> <title>Do lipid labs need to be fasting?</title> <deck/> </itemMeta> <itemContent> <p>When I worked as a scribe prior to starting medical school, it was commonplace for patients to have fasting labs. I always felt terrible for the patients we saw late in the afternoon that had somehow fasted all day. For many other patients, there was the challenge of finding a time when they could return to have fasting labs drawn. </p> <p><span class="tag metaDescription">While in medical school, I have seen the transition of my preceptors’ recommendations, where it seems patients can now have nonfasting labs.</span> However, I have still observed instances when patients need to have fasting labs. We can look at an example case to better understand when and why patients do and do not need to fast prior to having their lipids checked. [[{"fid":"289532","view_mode":"medstat_image_flush_right","fields":{"format":"medstat_image_flush_right","field_file_image_alt_text[und][0][value]":"Kendall Ervin, fourth-year medical student at the University of Washington, Seattle","field_file_image_credit[und][0][value]":"","field_file_image_caption[und][0][value]":"Kendall Ervin"},"type":"media","attributes":{"class":"media-element file-medstat_image_flush_right"}}]]<br/><br/></p> <h2>Case</h2> <p>A 57-year-old woman presents for an annual wellness visit. She has been healthy this past year with no new concerns. Her blood pressure has been well controlled, and she continues on a statin for hyperlipidemia. She is due for annual labs. She ate breakfast this morning. Which of the following do you recommend?<br/><br/>A. Obtain lipids with her other blood work now.<br/><br/>B. Have her return tomorrow to obtain fasting labs.<br/><br/>In this situation, A is the correct answer. The patient is due for routine screening labs and there are no current indications that fasting labs are necessary.</p> <h2>Studies of fasting vs. nonfasting lipids</h2> <p>Sidhu and Naugler performed a cross-sectional analysis comparing lipid values at fasting intervals of 1 hour to 16 hours.<sup>1</sup> They found the mean total cholesterol and HDL cholesterol values differed by greater than 2%. For LDL cholesterol, the values differed by less than 10% and triglycerides values differed by less than 20%. With this information, the researchers concluded fasting for routine lipids is generally unnecessary. </p> <p>Mora and colleagues performed a post hoc prospective follow-up of a randomized control [[{"fid":"248251","view_mode":"medstat_image_flush_right","fields":{"format":"medstat_image_flush_right","field_file_image_alt_text[und][0][value]":"Dr. Douglas S. Paauw, University of Washington, Seattle","field_file_image_credit[und][0][value]":"","field_file_image_caption[und][0][value]":"Dr. Douglas S. Paauw"},"type":"media","attributes":{"class":"media-element file-medstat_image_flush_right"}}]]trial to assess if nonfasting lipid measurements could cause misclassification of cardiovascular risk assessment.<sup>2</sup> Based on 8,270 participants, coronary events associated with fasting vs. nonfasting lipid values were similar when adjusted hazard ratios were compared. They also found an agreement of 94.8% when classifying participants into ASCVD risk categories for fasting and nonfasting lipid values. These outcomes led them to support the use of nonfasting lipid labs for routine cardiovascular risk assessment.<br/><br/>Rahman and colleagues performed a systematic review and found the use of nonfasting lipid values can reliably determine statin management in most situations.<sup>3</sup> Circumstances where fasting labs should be used are if patients have a genetic dyslipidemia, if patients have severe hypertriglyceridemia (greater than 500 mg/dL), and if patients have pancreatitis. Triglyceride values fluctuate the most between the fasting and nonfasting state as seen above from Sidhu and Naugler. This could impact triglyceride disorder management and the accuracy of LDL cholesterol estimation (calculated by the Friedewald equation: LDL cholesterol = total cholesterol – HDL cholesterol – triglycerides/5 in mg/dL).<sup>3</sup></p> <h2>Benefits of nonfasting lipid labs</h2> <p>There are many benefits of nonfasting labs. For the patients, they do not have to come to their appointments hungry, we can reduce the risk of hypoglycemia for those with diabetes, and they do not have come back at a later date if they ate something earlier in the day. </p> <p>For the lab, we can improve efficiency and decrease early morning congestion when patients typically come in for fasting labs. <br/><br/>Lastly, for the provider, nonfasting labs can improve workflow and help decrease the number of patients lost to follow-up who were unable to complete fasting labs the same day as their appointment. <br/><br/></p> <h2>Summary</h2> <p>Patients do not need to fast prior to having lipid levels drawn for routine screening. Fasting labs should be considered for patients who have a genetic dyslipidemia or if there is concern for hypertriglyceridemia. </p> <p>Per the ACC/AHA guidelines, nonfasting lipids can be used to assess ASCVD risk and to establish a baseline LDL cholesterol in adults 20 years and older. If a patient has nonfasting triglycerides greater than 400 mg/dL, repeat fasting lipids should be drawn to assess fasting triglycerides and to establish a baseline LDL cholesterol.<sup>4</sup> <br/><br/></p> <p> <em>Ms. Ervin is a fourth-year medical student at the University of Washington, Seattle. She has no conflicts to disclose. Dr. Paauw is professor of medicine in the division of general internal medicine at the University of Washington, and he serves as third-year medical student clerkship director at the university. He is a member of the editorial advisory board of Internal Medicine News. Dr. Paauw has no conflicts to disclose. Contact him at <span class="Hyperlink"><a href="mailto:imnews%40mdedge.com?subject=">imnews@mdedge.com</a></span>.</em> </p> <h2>References</h2> <p>1. Rahman F et al. <span class="Hyperlink"><a href="https://pubmed.ncbi.nlm.nih.gov/29455255/">Curr Atheroscler Rep. 2018;20(3):14</a></span>. Published 2018 Feb 17.<br/><br/>2. Mora S et al. <span class="Hyperlink"><a href="https://pubmed.ncbi.nlm.nih.gov/31135812/">JAMA Intern Med. 2019;179(7):898-905</a></span>.<br/><br/>3. Sidhu D and Naugler C. <span class="Hyperlink"><a href="https://pubmed.ncbi.nlm.nih.gov/23147400/">Arch Intern Med. 2012;172(22):1707-10</a></span>.<br/><br/>4. Hoover LE. <span class="Hyperlink"><a href="https://pubmed.ncbi.nlm.nih.gov/31038893/">Am Fam Physician. 2019 May 1;99(9):589-91</a></span>.</p> </itemContent> </newsItem> <newsItem> <itemMeta> <itemRole>teaser</itemRole> <itemClass>text</itemClass> <title/> <deck/> </itemMeta> <itemContent> </itemContent> </newsItem> </itemSet></root>
Disallow All Ads
Content Gating
No Gating (article Unlocked/Free)
Alternative CME
Disqus Comments
Default
Use ProPublica
Hide sidebar & use full width
render the right sidebar.
Conference Recap Checkbox
Not Conference Recap
Clinical Edge
Display the Slideshow in this Article
Medscape Article
Display survey writer
Reuters content
Disable Inline Native ads
WebMD Article

Is hepatitis C an STI?

Article Type
Changed
Tue, 06/21/2022 - 15:37

A 32-year-old woman had sex with a man she met while on vacation 6 weeks ago. She was intoxicated at the time and does not know much about the person. She recalls having engaged in vaginal intercourse without a condom. She does not have any symptoms.

She previously received baseline lab testing per Centers for Disease Control and Prevention guidelines 2 years ago with a negative HIV test and negative hepatitis C test. She asks for testing for STIs. What would you recommend?

Paauw_Doug2_web.jpg
Dr. Paauw

A. HIV, hepatitis C, gonorrhea, chlamydia, and human papillomavirus

B. HIV, hepatitis C, gonorrhea, chlamydia, and herpes simplex virus

C. HIV, hepatitis C, gonorrhea, and chlamydia

D. HIV, gonorrhea, and chlamydia

E. Gonorrhea and chlamydia

HIV risk estimate

The most practical answer is E, check for gonorrhea and chlamydia. Many protocols in place for evaluating people for STIs will test for hepatitis C as well as HIV with single exposures. In this column, we will look at the lack of evidence of heterosexual sexual transmission of hepatitis C.

In regards to HIV risk, the estimated risk of transmission male to female from an HIV-infected individual is 0.08% per sexual encounter.1 The prevalence in the United States – where HIV occurs in about 0.5% of the adult population – was used to estimate the risk of a person with unknown HIV status acquiring HIV. The calculated risk from one sexual encounter would be 0.0004 (1 in 250,000).
 

Studies of hepatitis C transmission

Tahan and colleagues did a prospective study of 600 heterosexual couples where one partner had hepatitis C and the other didn’t. Over a mean of 3 years of follow-up, none of the seronegative spouses developed hepatitis C.2

Terrault and colleagues completed a cross-sectional study of hepatitis C virus (HCV)–positive individuals and their monogamous heterosexual partners to evaluate risk of sexual transmission of HCV.3 Based on 8,377 person-years of follow-up, the estimated maximum transmission rate was 0.07%/year, which was about 1/190,000 sexual contacts. No specific sexual practices were associated with transmission. The authors of this study concurred with CDC recommendations that persons with HCV infection in long-term monogamous relationships need not change their sexual practices.4

Vandelli and colleagues followed 776 heterosexual partners of HCV-infected individuals over 10 years.5 None of the couples reported condom use. Over the follow up period, three HCV infections occurred, but based on discordance of the typing of viral isolates, sexual transmission was excluded.

Jin and colleagues completed a systematic review of studies looking at possible sexual transmission of HCV in gay and bisexual men.6 HIV-positive men had a HCV incidence of 6.4 per 1,000 person-years, compared with 0.4 per 1000 person-years in HIV-negative men. The authors discussed several possible causes for increased transmission risk in HIV-infected individuals including coexisting STIs and higher HCV viral load in semen of HIV-infected individuals, as well as lower immunity.
 

Summary

In hepatitis C–discordant heterosexual couples, hepatitis C does not appear to be sexually transmitted.

The risk of sexual transmission of hepatitis C to non–HIV-infected individuals appears to be exceedingly low.

Many thanks to Hunter Handsfield, MD, for suggesting this topic and sharing supporting articles.

Dr. Paauw is professor of medicine in the division of general internal medicine at the University of Washington, Seattle, and he serves as third-year medical student clerkship director at the University of Washington. He is a member of the editorial advisory board of Internal Medicine News. Dr. Paauw has no conflicts to disclose. Contact him at imnews@mdedge.com.

1. Boily MC et al. Lancet Infect Dis. 2009 Feb;9(2):118-29.

2. Tahan V et al. Am J Gastroenterol. 2005;100:821-4.

3. Terrault NA et al. Hepatology. 2013;57:881-9

4. Centers for Disease Control and Prevention. MMWR Recomm Rep. 1998;47:1-38.

5. Vandelli C et al. Am J Gastroenterol. 2004;99:855-9.

6. Jin F et al. Sexual Health.2017;14:28-41.

Publications
Topics
Sections

A 32-year-old woman had sex with a man she met while on vacation 6 weeks ago. She was intoxicated at the time and does not know much about the person. She recalls having engaged in vaginal intercourse without a condom. She does not have any symptoms.

She previously received baseline lab testing per Centers for Disease Control and Prevention guidelines 2 years ago with a negative HIV test and negative hepatitis C test. She asks for testing for STIs. What would you recommend?

Paauw_Doug2_web.jpg
Dr. Paauw

A. HIV, hepatitis C, gonorrhea, chlamydia, and human papillomavirus

B. HIV, hepatitis C, gonorrhea, chlamydia, and herpes simplex virus

C. HIV, hepatitis C, gonorrhea, and chlamydia

D. HIV, gonorrhea, and chlamydia

E. Gonorrhea and chlamydia

HIV risk estimate

The most practical answer is E, check for gonorrhea and chlamydia. Many protocols in place for evaluating people for STIs will test for hepatitis C as well as HIV with single exposures. In this column, we will look at the lack of evidence of heterosexual sexual transmission of hepatitis C.

In regards to HIV risk, the estimated risk of transmission male to female from an HIV-infected individual is 0.08% per sexual encounter.1 The prevalence in the United States – where HIV occurs in about 0.5% of the adult population – was used to estimate the risk of a person with unknown HIV status acquiring HIV. The calculated risk from one sexual encounter would be 0.0004 (1 in 250,000).
 

Studies of hepatitis C transmission

Tahan and colleagues did a prospective study of 600 heterosexual couples where one partner had hepatitis C and the other didn’t. Over a mean of 3 years of follow-up, none of the seronegative spouses developed hepatitis C.2

Terrault and colleagues completed a cross-sectional study of hepatitis C virus (HCV)–positive individuals and their monogamous heterosexual partners to evaluate risk of sexual transmission of HCV.3 Based on 8,377 person-years of follow-up, the estimated maximum transmission rate was 0.07%/year, which was about 1/190,000 sexual contacts. No specific sexual practices were associated with transmission. The authors of this study concurred with CDC recommendations that persons with HCV infection in long-term monogamous relationships need not change their sexual practices.4

Vandelli and colleagues followed 776 heterosexual partners of HCV-infected individuals over 10 years.5 None of the couples reported condom use. Over the follow up period, three HCV infections occurred, but based on discordance of the typing of viral isolates, sexual transmission was excluded.

Jin and colleagues completed a systematic review of studies looking at possible sexual transmission of HCV in gay and bisexual men.6 HIV-positive men had a HCV incidence of 6.4 per 1,000 person-years, compared with 0.4 per 1000 person-years in HIV-negative men. The authors discussed several possible causes for increased transmission risk in HIV-infected individuals including coexisting STIs and higher HCV viral load in semen of HIV-infected individuals, as well as lower immunity.
 

Summary

In hepatitis C–discordant heterosexual couples, hepatitis C does not appear to be sexually transmitted.

The risk of sexual transmission of hepatitis C to non–HIV-infected individuals appears to be exceedingly low.

Many thanks to Hunter Handsfield, MD, for suggesting this topic and sharing supporting articles.

Dr. Paauw is professor of medicine in the division of general internal medicine at the University of Washington, Seattle, and he serves as third-year medical student clerkship director at the University of Washington. He is a member of the editorial advisory board of Internal Medicine News. Dr. Paauw has no conflicts to disclose. Contact him at imnews@mdedge.com.

1. Boily MC et al. Lancet Infect Dis. 2009 Feb;9(2):118-29.

2. Tahan V et al. Am J Gastroenterol. 2005;100:821-4.

3. Terrault NA et al. Hepatology. 2013;57:881-9

4. Centers for Disease Control and Prevention. MMWR Recomm Rep. 1998;47:1-38.

5. Vandelli C et al. Am J Gastroenterol. 2004;99:855-9.

6. Jin F et al. Sexual Health.2017;14:28-41.

A 32-year-old woman had sex with a man she met while on vacation 6 weeks ago. She was intoxicated at the time and does not know much about the person. She recalls having engaged in vaginal intercourse without a condom. She does not have any symptoms.

She previously received baseline lab testing per Centers for Disease Control and Prevention guidelines 2 years ago with a negative HIV test and negative hepatitis C test. She asks for testing for STIs. What would you recommend?

Paauw_Doug2_web.jpg
Dr. Paauw

A. HIV, hepatitis C, gonorrhea, chlamydia, and human papillomavirus

B. HIV, hepatitis C, gonorrhea, chlamydia, and herpes simplex virus

C. HIV, hepatitis C, gonorrhea, and chlamydia

D. HIV, gonorrhea, and chlamydia

E. Gonorrhea and chlamydia

HIV risk estimate

The most practical answer is E, check for gonorrhea and chlamydia. Many protocols in place for evaluating people for STIs will test for hepatitis C as well as HIV with single exposures. In this column, we will look at the lack of evidence of heterosexual sexual transmission of hepatitis C.

In regards to HIV risk, the estimated risk of transmission male to female from an HIV-infected individual is 0.08% per sexual encounter.1 The prevalence in the United States – where HIV occurs in about 0.5% of the adult population – was used to estimate the risk of a person with unknown HIV status acquiring HIV. The calculated risk from one sexual encounter would be 0.0004 (1 in 250,000).
 

Studies of hepatitis C transmission

Tahan and colleagues did a prospective study of 600 heterosexual couples where one partner had hepatitis C and the other didn’t. Over a mean of 3 years of follow-up, none of the seronegative spouses developed hepatitis C.2

Terrault and colleagues completed a cross-sectional study of hepatitis C virus (HCV)–positive individuals and their monogamous heterosexual partners to evaluate risk of sexual transmission of HCV.3 Based on 8,377 person-years of follow-up, the estimated maximum transmission rate was 0.07%/year, which was about 1/190,000 sexual contacts. No specific sexual practices were associated with transmission. The authors of this study concurred with CDC recommendations that persons with HCV infection in long-term monogamous relationships need not change their sexual practices.4

Vandelli and colleagues followed 776 heterosexual partners of HCV-infected individuals over 10 years.5 None of the couples reported condom use. Over the follow up period, three HCV infections occurred, but based on discordance of the typing of viral isolates, sexual transmission was excluded.

Jin and colleagues completed a systematic review of studies looking at possible sexual transmission of HCV in gay and bisexual men.6 HIV-positive men had a HCV incidence of 6.4 per 1,000 person-years, compared with 0.4 per 1000 person-years in HIV-negative men. The authors discussed several possible causes for increased transmission risk in HIV-infected individuals including coexisting STIs and higher HCV viral load in semen of HIV-infected individuals, as well as lower immunity.
 

Summary

In hepatitis C–discordant heterosexual couples, hepatitis C does not appear to be sexually transmitted.

The risk of sexual transmission of hepatitis C to non–HIV-infected individuals appears to be exceedingly low.

Many thanks to Hunter Handsfield, MD, for suggesting this topic and sharing supporting articles.

Dr. Paauw is professor of medicine in the division of general internal medicine at the University of Washington, Seattle, and he serves as third-year medical student clerkship director at the University of Washington. He is a member of the editorial advisory board of Internal Medicine News. Dr. Paauw has no conflicts to disclose. Contact him at imnews@mdedge.com.

1. Boily MC et al. Lancet Infect Dis. 2009 Feb;9(2):118-29.

2. Tahan V et al. Am J Gastroenterol. 2005;100:821-4.

3. Terrault NA et al. Hepatology. 2013;57:881-9

4. Centers for Disease Control and Prevention. MMWR Recomm Rep. 1998;47:1-38.

5. Vandelli C et al. Am J Gastroenterol. 2004;99:855-9.

6. Jin F et al. Sexual Health.2017;14:28-41.

Publications
Publications
Topics
Article Type
Sections
Teambase XML
<?xml version="1.0" encoding="UTF-8"?>
<!--$RCSfile: InCopy_agile.xsl,v $ $Revision: 1.35 $-->
<!--$RCSfile: drupal.xsl,v $ $Revision: 1.7 $-->
<root generator="drupal.xsl" gversion="1.7"> <header> <fileName>158525</fileName> <TBEID>0C043419.SIG</TBEID> <TBUniqueIdentifier>MD_0C043419</TBUniqueIdentifier> <newsOrJournal>News</newsOrJournal> <publisherName>Frontline Medical Communications</publisherName> <storyname>Is Hepatitis C an STI?</storyname> <articleType>353</articleType> <TBLocation>QC Done-All Pubs</TBLocation> <QCDate>20220615T114846</QCDate> <firstPublished>20220615T163114</firstPublished> <LastPublished>20220615T163114</LastPublished> <pubStatus qcode="stat:"/> <embargoDate/> <killDate/> <CMSDate>20220615T163114</CMSDate> <articleSource/> <facebookInfo/> <meetingNumber/> <byline/> <bylineText>DOUGLAS S. PAAUW, MD</bylineText> <bylineFull>DOUGLAS S. PAAUW, MD</bylineFull> <bylineTitleText>MDedge News</bylineTitleText> <USOrGlobal/> <wireDocType/> <newsDocType>Opinion</newsDocType> <journalDocType/> <linkLabel/> <pageRange/> <citation/> <quizID/> <indexIssueDate/> <itemClass qcode="ninat:text"/> <provider qcode="provider:imng"> <name>IMNG Medical Media</name> <rightsInfo> <copyrightHolder> <name>Frontline Medical News</name> </copyrightHolder> <copyrightNotice>Copyright (c) 2015 Frontline Medical News, a Frontline Medical Communications Inc. company. All rights reserved. This material may not be published, broadcast, copied, or otherwise reproduced or distributed without the prior written permission of Frontline Medical Communications Inc.</copyrightNotice> </rightsInfo> </provider> <abstract/> <metaDescription>A 32-year-old woman had sex with a man she met while on vacation 6 weeks ago. She was intoxicated at the time and does not know much about the person. She recal</metaDescription> <articlePDF/> <teaserImage>241147</teaserImage> <teaser>Which sexually transmitted infection tests should this patient receive? </teaser> <title>Is hepatitis C an STI?</title> <deck/> <disclaimer/> <AuthorList/> <articleURL/> <doi/> <pubMedID/> <publishXMLStatus/> <publishXMLVersion>1</publishXMLVersion> <useEISSN>0</useEISSN> <urgency/> <pubPubdateYear/> <pubPubdateMonth/> <pubPubdateDay/> <pubVolume/> <pubNumber/> <wireChannels/> <primaryCMSID/> <CMSIDs/> <keywords/> <seeAlsos/> <publications_g> <publicationData> <publicationCode>idprac</publicationCode> <pubIssueName/> <pubArticleType/> <pubTopics/> <pubCategories/> <pubSections/> </publicationData> <publicationData> <publicationCode>fp</publicationCode> <pubIssueName/> <pubArticleType/> <pubTopics/> <pubCategories/> <pubSections/> </publicationData> <publicationData> <publicationCode>im</publicationCode> <pubIssueName/> <pubArticleType/> <pubTopics/> <pubCategories/> <pubSections/> </publicationData> </publications_g> <publications> <term>20</term> <term>15</term> <term canonical="true">21</term> </publications> <sections> <term canonical="true">28225</term> <term>41022</term> <term>52</term> </sections> <topics> <term>280</term> <term canonical="true">234</term> <term>50729</term> <term>50347</term> <term>318</term> <term>314</term> </topics> <links> <link> <itemClass qcode="ninat:picture"/> <altRep contenttype="image/jpeg">images/2400b8e1.jpg</altRep> <description role="drol:caption">Dr. Paauw</description> <description role="drol:credit"/> </link> </links> </header> <itemSet> <newsItem> <itemMeta> <itemRole>Main</itemRole> <itemClass>text</itemClass> <title>Is hepatitis C an STI?</title> <deck/> </itemMeta> <itemContent> <p>A 32-year-old woman had sex with a man she met while on vacation 6 weeks ago. She was intoxicated at the time and does not know much about the person. She recalls having engaged in vaginal intercourse without a condom. She does not have any symptoms. </p> <p>She previously received baseline lab testing per Centers for Disease Control and Prevention guidelines 2 years ago with a negative HIV test and negative hepatitis C test. She asks for testing for STIs. What would you recommend?<br/><br/>[[{"fid":"241147","view_mode":"medstat_image_flush_left","fields":{"format":"medstat_image_flush_left","field_file_image_alt_text[und][0][value]":"Dr. Paauw","field_file_image_credit[und][0][value]":"","field_file_image_caption[und][0][value]":"Dr. Paauw"},"type":"media","attributes":{"class":"media-element file-medstat_image_flush_left"}}]]</p> <p><strong>A.</strong> HIV, hepatitis C, gonorrhea, chlamydia, and human papillomavirus <br/><br/><strong>B.</strong> HIV, hepatitis C, gonorrhea, chlamydia, and herpes simplex virus<br/><br/><strong>C. </strong>HIV, hepatitis C, gonorrhea, and chlamydia<br/><br/><strong>D.</strong> HIV, gonorrhea, and chlamydia<br/><br/><strong>E.</strong> Gonorrhea and chlamydia</p> <h2>HIV risk estimate</h2> <p>The most practical answer is E, check for gonorrhea and chlamydia. Many protocols in place for evaluating people for STIs will test for hepatitis C as well as HIV with single exposures. In this column, we will look at the lack of evidence of heterosexual sexual transmission of hepatitis C. </p> <p>In regards to HIV risk, the estimated risk of transmission male to female from an HIV-infected individual is 0.08% per sexual encounter.<sup>1</sup> The prevalence in the United States – where HIV occurs in about 0.5% of the adult population – was used to estimate the risk of a person with unknown HIV status acquiring HIV. The calculated risk from one sexual encounter would be 0.0004 (1 in 250,000).<br/><br/></p> <h2>Studies of hepatitis C transmission</h2> <p>Tahan and colleagues did a prospective study of 600 heterosexual couples where one partner had hepatitis C and the other didn’t. Over a mean of 3 years of follow-up, none of the seronegative spouses developed hepatitis C.<sup>2</sup> </p> <p>Terrault and colleagues completed a cross-sectional study of hepatitis C virus (HCV)–positive individuals and their monogamous heterosexual partners to evaluate risk of sexual transmission of HCV.<sup>3</sup> Based on 8,377 person-years of follow-up, the estimated maximum transmission rate was 0.07%/year, which was about 1/190,000 sexual contacts. No specific sexual practices were associated with transmission. The authors of this study concurred with CDC recommendations that persons with HCV infection in long-term monogamous relationships need not change their sexual practices.<sup>4</sup> <br/><br/>Vandelli and colleagues followed 776 heterosexual partners of HCV-infected individuals over 10 years.<sup>5</sup> None of the couples reported condom use. Over the follow up period, threw HCV infections occurred, but based on discordance of the typing of viral isolates, sexual transmission was excluded. <br/><br/>Jin and colleagues completed a systematic review of studies looking at possible sexual transmission of HCV in gay and bisexual men.<sup>6</sup> HIV-positive men had a HCV incidence of 6.4 per 1,000 person-years, compared with 0.4 per 1000 person-years in HIV-negative men. The authors discussed several possible causes for increased transmission risk in HIV-infected individuals including coexisting STIs and higher HCV viral load in semen of HIV-infected individuals, as well as lower immunity.<br/><br/></p> <h2>Summary</h2> <p>In hepatitis C–discordant heterosexual couples, hepatitis C does not appear to be sexually transmitted.</p> <p>The risk of sexual transmission of hepatitis C to non–HIV-infected individuals appears to be exceedingly low.</p> <p> <em>Many thanks to Hunter Handsfield, MD, for suggesting this topic and sharing supporting articles.</em> </p> <p> <em>Dr. Paauw is professor of medicine in the division of general internal medicine at the University of Washington, Seattle, and he serves as third-year medical student clerkship director at the University of Washington. He is a member of the editorial advisory board of Internal Medicine News. Dr. Paauw has no conflicts to disclose. Contact him at imnews@mdedge.com.</em> </p> <p>1. Boily MC et al. Lancet Infect Dis. 2009 Feb;9(2):118-29.<br/><br/>2. Tahan V et al. Am J Gastroenterol. 2005;100:821-4.<br/><br/>3. Terrault NA et al. Hepatology. 2013;57:881-9<br/><br/>4. Centers for Disease Control and Prevention. MMWR Recomm Rep. 1998;47:1-38.<br/><br/>5. Vandelli C et al. Am J Gastroenterol. 2004;99:855-9.<br/><br/>6. Jin F et al. Sexual Health.2017;14:28-41.</p> </itemContent> </newsItem> <newsItem> <itemMeta> <itemRole>teaser</itemRole> <itemClass>text</itemClass> <title/> <deck/> </itemMeta> <itemContent> </itemContent> </newsItem> </itemSet></root>
Disallow All Ads
Content Gating
No Gating (article Unlocked/Free)
Alternative CME
Disqus Comments
Default
Use ProPublica
Hide sidebar & use full width
render the right sidebar.
Conference Recap Checkbox
Not Conference Recap
Clinical Edge
Display the Slideshow in this Article
Medscape Article
Display survey writer
Reuters content
Disable Inline Native ads
WebMD Article

What is the most likely cause of this patient’s fever?

Article Type
Changed
Wed, 08/25/2021 - 16:42

A 63-year-old man undergoes cardiac bypass surgery. He is able to be extubated at 8 hours. The next morning he has a fever to 38.5° C His exam shows no redness at the surgical site, or at his IV sites. His lung exam is unremarkable. His urinalysis is without white blood cells. His white blood cell count is 8,500, and his chest x-ray shows atelectasis without other abnormalities.

Paauw_Doug_SEATTLE_2019_web2.jpg
Dr. Douglas S. Paauw

What is the most likely cause of this patient’s fever? The research papers cited below suggest what it isn’t and what it might be.

One of the earliest things I was taught in my clinical years were the causes of postoperative fever, or the 5Ws, which are wind, water, wound, walk, and wonder drug.

Atelectasis was touted as the cause of early postoperative fever. This became clear fact in my medical student mind, not something that I had ever questioned.  But investigation into whether there is evidence of this shows it is only a myth. In actuality, there is scant evidence, if any, for atelectasis causing fever. Frequently, no cause of postoperative fever has been found, despite aggressive attempts to look for one.

What the research says

Fanning and colleagues prospectively looked at 537 women who were undergoing major gynecologic surgery.1 Postoperative fever occurred in 211 of them. In 92% of these patients, no cause for fever was found.

Atelectasis is frequently seen postoperatively. Schlenker and colleagues reported that, in patients with postoperative atelectasis, temperature elevation on the first postoperative day was directly related to the degree of atelectasis, but the white blood cell count elevation was inversely related.2

In this study, atelectasis was diagnosed by auscultation, with chest x-rays ordered at the discretion of the physician. There was little correlation with the auscultatory findings and presence or absence of atelectasis in the patients who did receive chest x-rays.

Engoren did a study to prospectively evaluate 100 postoperative patients with daily chest x-rays and continuous temperature monitoring.3 Results from the day of surgery (day 0) to the second postoperative day showed an increase in presence of atelectasis from 43% on the day of surgery to 79% by day 2.

Fever, defined as temperature greater than 38° C, fell from 37% on the day of surgery to 17% by day 2. Engoren found no association between fever and degree of atelectasis.

Mavros and colleagues did a comprehensive review to determine whether there was evidence to support atelectasis causing fever.4 They concluded that there was no clinical evidence supporting the concept that atelectasis is associated with early postoperative fever.
 

A possible cause of fever

Mavros and colleagues’ paper suggested that early postoperative fever was caused by stress derived by surgery, which can increase the patient’s interleukin-6 levels and thermostatic set point. This was demonstrated in a small study by Wortel and colleagues, who measured IL-6 levels in the portal and peripheral blood of patients following pancreaticoduodenectomy.5 They found IL-6 levels correlated strongly with peak body temperature.

In conclusion, atelectasis is not a well-established cause of postoperative fever.

Dr. Paauw is professor of medicine in the division of general internal medicine at the University of Washington, Seattle, and he serves as third-year medical student clerkship director at the University of Washington. He is a member of the editorial advisory board of Internal Medicine News. Dr. Paauw has no conflicts to disclose. Contact him at imnews@mdedge.com.

References

1. Fanning J et al. Infect Dis Obstet Gynecol. 1998; 6(6):252-5 .

2. Schlenker JD and Hubay CA. Arch Surg 1973;107:846-50

3. Engoren M. Chest. 1995;107(1):81-4 .

4. Michael N et al. Chest. 2011;140(2):418-24

5. Wortel CH et al. Surgery. 1993;114(3):564-70 .

Publications
Topics
Sections

A 63-year-old man undergoes cardiac bypass surgery. He is able to be extubated at 8 hours. The next morning he has a fever to 38.5° C His exam shows no redness at the surgical site, or at his IV sites. His lung exam is unremarkable. His urinalysis is without white blood cells. His white blood cell count is 8,500, and his chest x-ray shows atelectasis without other abnormalities.

Paauw_Doug_SEATTLE_2019_web2.jpg
Dr. Douglas S. Paauw

What is the most likely cause of this patient’s fever? The research papers cited below suggest what it isn’t and what it might be.

One of the earliest things I was taught in my clinical years were the causes of postoperative fever, or the 5Ws, which are wind, water, wound, walk, and wonder drug.

Atelectasis was touted as the cause of early postoperative fever. This became clear fact in my medical student mind, not something that I had ever questioned.  But investigation into whether there is evidence of this shows it is only a myth. In actuality, there is scant evidence, if any, for atelectasis causing fever. Frequently, no cause of postoperative fever has been found, despite aggressive attempts to look for one.

What the research says

Fanning and colleagues prospectively looked at 537 women who were undergoing major gynecologic surgery.1 Postoperative fever occurred in 211 of them. In 92% of these patients, no cause for fever was found.

Atelectasis is frequently seen postoperatively. Schlenker and colleagues reported that, in patients with postoperative atelectasis, temperature elevation on the first postoperative day was directly related to the degree of atelectasis, but the white blood cell count elevation was inversely related.2

In this study, atelectasis was diagnosed by auscultation, with chest x-rays ordered at the discretion of the physician. There was little correlation with the auscultatory findings and presence or absence of atelectasis in the patients who did receive chest x-rays.

Engoren did a study to prospectively evaluate 100 postoperative patients with daily chest x-rays and continuous temperature monitoring.3 Results from the day of surgery (day 0) to the second postoperative day showed an increase in presence of atelectasis from 43% on the day of surgery to 79% by day 2.

Fever, defined as temperature greater than 38° C, fell from 37% on the day of surgery to 17% by day 2. Engoren found no association between fever and degree of atelectasis.

Mavros and colleagues did a comprehensive review to determine whether there was evidence to support atelectasis causing fever.4 They concluded that there was no clinical evidence supporting the concept that atelectasis is associated with early postoperative fever.
 

A possible cause of fever

Mavros and colleagues’ paper suggested that early postoperative fever was caused by stress derived by surgery, which can increase the patient’s interleukin-6 levels and thermostatic set point. This was demonstrated in a small study by Wortel and colleagues, who measured IL-6 levels in the portal and peripheral blood of patients following pancreaticoduodenectomy.5 They found IL-6 levels correlated strongly with peak body temperature.

In conclusion, atelectasis is not a well-established cause of postoperative fever.

Dr. Paauw is professor of medicine in the division of general internal medicine at the University of Washington, Seattle, and he serves as third-year medical student clerkship director at the University of Washington. He is a member of the editorial advisory board of Internal Medicine News. Dr. Paauw has no conflicts to disclose. Contact him at imnews@mdedge.com.

References

1. Fanning J et al. Infect Dis Obstet Gynecol. 1998; 6(6):252-5 .

2. Schlenker JD and Hubay CA. Arch Surg 1973;107:846-50

3. Engoren M. Chest. 1995;107(1):81-4 .

4. Michael N et al. Chest. 2011;140(2):418-24

5. Wortel CH et al. Surgery. 1993;114(3):564-70 .

A 63-year-old man undergoes cardiac bypass surgery. He is able to be extubated at 8 hours. The next morning he has a fever to 38.5° C His exam shows no redness at the surgical site, or at his IV sites. His lung exam is unremarkable. His urinalysis is without white blood cells. His white blood cell count is 8,500, and his chest x-ray shows atelectasis without other abnormalities.

Paauw_Doug_SEATTLE_2019_web2.jpg
Dr. Douglas S. Paauw

What is the most likely cause of this patient’s fever? The research papers cited below suggest what it isn’t and what it might be.

One of the earliest things I was taught in my clinical years were the causes of postoperative fever, or the 5Ws, which are wind, water, wound, walk, and wonder drug.

Atelectasis was touted as the cause of early postoperative fever. This became clear fact in my medical student mind, not something that I had ever questioned.  But investigation into whether there is evidence of this shows it is only a myth. In actuality, there is scant evidence, if any, for atelectasis causing fever. Frequently, no cause of postoperative fever has been found, despite aggressive attempts to look for one.

What the research says

Fanning and colleagues prospectively looked at 537 women who were undergoing major gynecologic surgery.1 Postoperative fever occurred in 211 of them. In 92% of these patients, no cause for fever was found.

Atelectasis is frequently seen postoperatively. Schlenker and colleagues reported that, in patients with postoperative atelectasis, temperature elevation on the first postoperative day was directly related to the degree of atelectasis, but the white blood cell count elevation was inversely related.2

In this study, atelectasis was diagnosed by auscultation, with chest x-rays ordered at the discretion of the physician. There was little correlation with the auscultatory findings and presence or absence of atelectasis in the patients who did receive chest x-rays.

Engoren did a study to prospectively evaluate 100 postoperative patients with daily chest x-rays and continuous temperature monitoring.3 Results from the day of surgery (day 0) to the second postoperative day showed an increase in presence of atelectasis from 43% on the day of surgery to 79% by day 2.

Fever, defined as temperature greater than 38° C, fell from 37% on the day of surgery to 17% by day 2. Engoren found no association between fever and degree of atelectasis.

Mavros and colleagues did a comprehensive review to determine whether there was evidence to support atelectasis causing fever.4 They concluded that there was no clinical evidence supporting the concept that atelectasis is associated with early postoperative fever.
 

A possible cause of fever

Mavros and colleagues’ paper suggested that early postoperative fever was caused by stress derived by surgery, which can increase the patient’s interleukin-6 levels and thermostatic set point. This was demonstrated in a small study by Wortel and colleagues, who measured IL-6 levels in the portal and peripheral blood of patients following pancreaticoduodenectomy.5 They found IL-6 levels correlated strongly with peak body temperature.

In conclusion, atelectasis is not a well-established cause of postoperative fever.

Dr. Paauw is professor of medicine in the division of general internal medicine at the University of Washington, Seattle, and he serves as third-year medical student clerkship director at the University of Washington. He is a member of the editorial advisory board of Internal Medicine News. Dr. Paauw has no conflicts to disclose. Contact him at imnews@mdedge.com.

References

1. Fanning J et al. Infect Dis Obstet Gynecol. 1998; 6(6):252-5 .

2. Schlenker JD and Hubay CA. Arch Surg 1973;107:846-50

3. Engoren M. Chest. 1995;107(1):81-4 .

4. Michael N et al. Chest. 2011;140(2):418-24

5. Wortel CH et al. Surgery. 1993;114(3):564-70 .

Publications
Publications
Topics
Article Type
Sections
Disallow All Ads
Content Gating
No Gating (article Unlocked/Free)
Alternative CME
Disqus Comments
Default
Use ProPublica
Hide sidebar & use full width
render the right sidebar.
Conference Recap Checkbox
Not Conference Recap
Clinical Edge
Display the Slideshow in this Article
Medscape Article
Display survey writer
Reuters content
Disable Inline Native ads
WebMD Article

Does optimal iron absorption include vitamin C?

Article Type
Changed
Wed, 07/14/2021 - 14:17

 

A 46-year-old woman presents with fatigue. She reports that she has had unusually heavy periods for the past 6 months. Her blood work shows a hematocrit level of 32, a mean corpuscular volume of 77, a platelet count of 390,000, and a ferritin level of 5.

Paauw_Doug_SEATTLE_2019_web.jpg
Dr. Douglas S. Paauw

What would you recommend for iron replacement?

A. FeSO4 325 mg three times a day with vitamin C

B. FeSO4 325 mg daily with vitamin C

C. FeSO4 325 mg every other day

Recommendations and supporting research

I think I would start with choice C, FeSO4 every other day.

Treatment of iron deficiency with oral iron has traditionally been done by giving 150-200 mg of elemental iron (which is equal to three 325 mg tablets of iron sulfate).1 This dosing regimen has considerable gastrointestinal side effects. Recent evidence has shown that iron absorption is diminished the more frequently it is given.

Stoffel and colleagues found that fractional iron absorption was higher in iron-deficient women who were given iron every other day, compared with those who received daily iron.2 They also found that the more frequently iron was administered, the higher the hepcidin levels were, and the lower the iron absorption.

Karacok and colleagues studied every other day iron versus daily iron for the treatment of iron-deficiency anemia of pregnancy.3 A total of 217 women completed randomization and participated in the study, with all women receiving 100 mg of elemental iron, either daily (111) or every other day (106). There was no significant difference in increase in ferritin levels, or hemoglobin increase between the groups. The daily iron group had more gastrointestinal symptoms (41.4%) than the every other day iron group (15.1%) (P < .0057).

Düzen Oflas and colleagues looked at the same question in nonpregnant women with iron deficiency anemia.4 Study patients either received 80 mg iron sulfate twice a day, 80 mg once a day, or 80 mg every other day. There was no statistically significant difference in hemoglobin improvement between groups, but the group that received twice a day dosing of iron had statistically significantly higher ferritin levels than the daily or every other day iron groups. This improvement in ferritin levels came at a cost, though, as 68% of patients in the twice daily iron group had gastrointestinal symptoms, compared with only 10% in the every other day iron group (P < .01).

Vitamin C is often recommended to be taken with iron to promote absorption. The evidence for this practice is scant, and dates back almost 50 years.5,6

Cook and Reddy found there was no significant difference in mean iron absorption among the three dietary periods studied in 12 patients despite a range of mean daily intakes of dietary vitamin C of 51-247 mg/d.7

Hunt and colleagues studied 25 non pregnant, healthy women with low ferritin levels.8 The women’s meals were supplemented with vitamin C (500 mg, three times a day) for 5 of the 10 weeks, in a double-blind, crossover design. Vitamin C supplementation did not lead to a difference in iron absorption, lab indices of iron deficiency, or the biological half-life of iron.

Li and colleagues looked at the effect of vitamin C supplementation on iron levels in women with iron deficiency anemia.9 A total of 440 women were recruited, with 432 completing the trial. Women were randomized to receive iron supplements plus vitamin C or iron supplements only. Their findings were that oral iron supplements alone were equivalent to oral iron supplements plus vitamin C in improving hemoglobin recovery and iron absorption.
 

Bottom line

Less frequent administration of iron supplements (every other day) is as effective as more frequent administration, with less GI symptoms. Also, adding vitamin C does not appear to improve absorption of iron supplements.

Dr. Paauw is professor of medicine in the division of general internal medicine at the University of Washington, Seattle, and he serves as third-year medical student clerkship director at the University of Washington. He is a member of the editorial advisory board of Internal Medicine News. Dr. Paauw has no conflicts to disclose. Contact him at imnews@mdedge.com.

References

1. 1. Fairbanks VF and Beutler E. Iron deficiency, in “Williams Textbook of Hematology, 6th ed.” (New York: McGraw-Hill, 2001).

2. Stoffel N et al. Lancet Haematology. 2017;4: e524-33.

3. Karakoc G et al. J Matern Fetal Neonatal Med. 2021 Apr 18:1-5

4. Düzen Oflas N et al. Intern Med J. 2020 Jul;50(7):854-8

5. Cook JD and Monsen ER. Am J Clin Nutr. 1977;30:235-41.

6. Hallberg L etal. Hum Nutr Appl Nutr. 1986;40: 97-113.

7. Cook JD and Reddy M. Am J Clin Nutr. 2001;73:93-8.

8. Hunt JR et al. Am J Clin Nutr. 1994 Jun;59(6):1381-5.

9. Li N et al. JAMA Netw Open. 2020 Nov 2;3(11):e2023644.

Publications
Topics
Sections

 

A 46-year-old woman presents with fatigue. She reports that she has had unusually heavy periods for the past 6 months. Her blood work shows a hematocrit level of 32, a mean corpuscular volume of 77, a platelet count of 390,000, and a ferritin level of 5.

Paauw_Doug_SEATTLE_2019_web.jpg
Dr. Douglas S. Paauw

What would you recommend for iron replacement?

A. FeSO4 325 mg three times a day with vitamin C

B. FeSO4 325 mg daily with vitamin C

C. FeSO4 325 mg every other day

Recommendations and supporting research

I think I would start with choice C, FeSO4 every other day.

Treatment of iron deficiency with oral iron has traditionally been done by giving 150-200 mg of elemental iron (which is equal to three 325 mg tablets of iron sulfate).1 This dosing regimen has considerable gastrointestinal side effects. Recent evidence has shown that iron absorption is diminished the more frequently it is given.

Stoffel and colleagues found that fractional iron absorption was higher in iron-deficient women who were given iron every other day, compared with those who received daily iron.2 They also found that the more frequently iron was administered, the higher the hepcidin levels were, and the lower the iron absorption.

Karacok and colleagues studied every other day iron versus daily iron for the treatment of iron-deficiency anemia of pregnancy.3 A total of 217 women completed randomization and participated in the study, with all women receiving 100 mg of elemental iron, either daily (111) or every other day (106). There was no significant difference in increase in ferritin levels, or hemoglobin increase between the groups. The daily iron group had more gastrointestinal symptoms (41.4%) than the every other day iron group (15.1%) (P < .0057).

Düzen Oflas and colleagues looked at the same question in nonpregnant women with iron deficiency anemia.4 Study patients either received 80 mg iron sulfate twice a day, 80 mg once a day, or 80 mg every other day. There was no statistically significant difference in hemoglobin improvement between groups, but the group that received twice a day dosing of iron had statistically significantly higher ferritin levels than the daily or every other day iron groups. This improvement in ferritin levels came at a cost, though, as 68% of patients in the twice daily iron group had gastrointestinal symptoms, compared with only 10% in the every other day iron group (P < .01).

Vitamin C is often recommended to be taken with iron to promote absorption. The evidence for this practice is scant, and dates back almost 50 years.5,6

Cook and Reddy found there was no significant difference in mean iron absorption among the three dietary periods studied in 12 patients despite a range of mean daily intakes of dietary vitamin C of 51-247 mg/d.7

Hunt and colleagues studied 25 non pregnant, healthy women with low ferritin levels.8 The women’s meals were supplemented with vitamin C (500 mg, three times a day) for 5 of the 10 weeks, in a double-blind, crossover design. Vitamin C supplementation did not lead to a difference in iron absorption, lab indices of iron deficiency, or the biological half-life of iron.

Li and colleagues looked at the effect of vitamin C supplementation on iron levels in women with iron deficiency anemia.9 A total of 440 women were recruited, with 432 completing the trial. Women were randomized to receive iron supplements plus vitamin C or iron supplements only. Their findings were that oral iron supplements alone were equivalent to oral iron supplements plus vitamin C in improving hemoglobin recovery and iron absorption.
 

Bottom line

Less frequent administration of iron supplements (every other day) is as effective as more frequent administration, with less GI symptoms. Also, adding vitamin C does not appear to improve absorption of iron supplements.

Dr. Paauw is professor of medicine in the division of general internal medicine at the University of Washington, Seattle, and he serves as third-year medical student clerkship director at the University of Washington. He is a member of the editorial advisory board of Internal Medicine News. Dr. Paauw has no conflicts to disclose. Contact him at imnews@mdedge.com.

References

1. 1. Fairbanks VF and Beutler E. Iron deficiency, in “Williams Textbook of Hematology, 6th ed.” (New York: McGraw-Hill, 2001).

2. Stoffel N et al. Lancet Haematology. 2017;4: e524-33.

3. Karakoc G et al. J Matern Fetal Neonatal Med. 2021 Apr 18:1-5

4. Düzen Oflas N et al. Intern Med J. 2020 Jul;50(7):854-8

5. Cook JD and Monsen ER. Am J Clin Nutr. 1977;30:235-41.

6. Hallberg L etal. Hum Nutr Appl Nutr. 1986;40: 97-113.

7. Cook JD and Reddy M. Am J Clin Nutr. 2001;73:93-8.

8. Hunt JR et al. Am J Clin Nutr. 1994 Jun;59(6):1381-5.

9. Li N et al. JAMA Netw Open. 2020 Nov 2;3(11):e2023644.

 

A 46-year-old woman presents with fatigue. She reports that she has had unusually heavy periods for the past 6 months. Her blood work shows a hematocrit level of 32, a mean corpuscular volume of 77, a platelet count of 390,000, and a ferritin level of 5.

Paauw_Doug_SEATTLE_2019_web.jpg
Dr. Douglas S. Paauw

What would you recommend for iron replacement?

A. FeSO4 325 mg three times a day with vitamin C

B. FeSO4 325 mg daily with vitamin C

C. FeSO4 325 mg every other day

Recommendations and supporting research

I think I would start with choice C, FeSO4 every other day.

Treatment of iron deficiency with oral iron has traditionally been done by giving 150-200 mg of elemental iron (which is equal to three 325 mg tablets of iron sulfate).1 This dosing regimen has considerable gastrointestinal side effects. Recent evidence has shown that iron absorption is diminished the more frequently it is given.

Stoffel and colleagues found that fractional iron absorption was higher in iron-deficient women who were given iron every other day, compared with those who received daily iron.2 They also found that the more frequently iron was administered, the higher the hepcidin levels were, and the lower the iron absorption.

Karacok and colleagues studied every other day iron versus daily iron for the treatment of iron-deficiency anemia of pregnancy.3 A total of 217 women completed randomization and participated in the study, with all women receiving 100 mg of elemental iron, either daily (111) or every other day (106). There was no significant difference in increase in ferritin levels, or hemoglobin increase between the groups. The daily iron group had more gastrointestinal symptoms (41.4%) than the every other day iron group (15.1%) (P < .0057).

Düzen Oflas and colleagues looked at the same question in nonpregnant women with iron deficiency anemia.4 Study patients either received 80 mg iron sulfate twice a day, 80 mg once a day, or 80 mg every other day. There was no statistically significant difference in hemoglobin improvement between groups, but the group that received twice a day dosing of iron had statistically significantly higher ferritin levels than the daily or every other day iron groups. This improvement in ferritin levels came at a cost, though, as 68% of patients in the twice daily iron group had gastrointestinal symptoms, compared with only 10% in the every other day iron group (P < .01).

Vitamin C is often recommended to be taken with iron to promote absorption. The evidence for this practice is scant, and dates back almost 50 years.5,6

Cook and Reddy found there was no significant difference in mean iron absorption among the three dietary periods studied in 12 patients despite a range of mean daily intakes of dietary vitamin C of 51-247 mg/d.7

Hunt and colleagues studied 25 non pregnant, healthy women with low ferritin levels.8 The women’s meals were supplemented with vitamin C (500 mg, three times a day) for 5 of the 10 weeks, in a double-blind, crossover design. Vitamin C supplementation did not lead to a difference in iron absorption, lab indices of iron deficiency, or the biological half-life of iron.

Li and colleagues looked at the effect of vitamin C supplementation on iron levels in women with iron deficiency anemia.9 A total of 440 women were recruited, with 432 completing the trial. Women were randomized to receive iron supplements plus vitamin C or iron supplements only. Their findings were that oral iron supplements alone were equivalent to oral iron supplements plus vitamin C in improving hemoglobin recovery and iron absorption.
 

Bottom line

Less frequent administration of iron supplements (every other day) is as effective as more frequent administration, with less GI symptoms. Also, adding vitamin C does not appear to improve absorption of iron supplements.

Dr. Paauw is professor of medicine in the division of general internal medicine at the University of Washington, Seattle, and he serves as third-year medical student clerkship director at the University of Washington. He is a member of the editorial advisory board of Internal Medicine News. Dr. Paauw has no conflicts to disclose. Contact him at imnews@mdedge.com.

References

1. 1. Fairbanks VF and Beutler E. Iron deficiency, in “Williams Textbook of Hematology, 6th ed.” (New York: McGraw-Hill, 2001).

2. Stoffel N et al. Lancet Haematology. 2017;4: e524-33.

3. Karakoc G et al. J Matern Fetal Neonatal Med. 2021 Apr 18:1-5

4. Düzen Oflas N et al. Intern Med J. 2020 Jul;50(7):854-8

5. Cook JD and Monsen ER. Am J Clin Nutr. 1977;30:235-41.

6. Hallberg L etal. Hum Nutr Appl Nutr. 1986;40: 97-113.

7. Cook JD and Reddy M. Am J Clin Nutr. 2001;73:93-8.

8. Hunt JR et al. Am J Clin Nutr. 1994 Jun;59(6):1381-5.

9. Li N et al. JAMA Netw Open. 2020 Nov 2;3(11):e2023644.

Publications
Publications
Topics
Article Type
Sections
Disallow All Ads
Content Gating
No Gating (article Unlocked/Free)
Alternative CME
Disqus Comments
Default
Use ProPublica
Hide sidebar & use full width
render the right sidebar.
Conference Recap Checkbox
Not Conference Recap
Clinical Edge
Display the Slideshow in this Article
Medscape Article
Display survey writer
Reuters content
Disable Inline Native ads
WebMD Article

Cellulitis treatment recommendations

Article Type
Changed
Thu, 02/25/2021 - 09:37

An obese 64-year-old man with type 2 diabetes presents with redness and warmth of his lower left leg.

Paauw_Doug_SEATTLE_2019_web.jpg
Dr. Douglas S. Paauw

He noticed discomfort today and saw that his left lower leg had redness and was warm. He does not recall scratches or injury to his leg. He has not had fever or chills. He has no other symptoms. His diabetes has been well controlled with diet and metformin.

On exam, his blood pressure is 120/70, pulse is 80, temperature is 37 degrees Celsius.

In the left lower extremity, the patient had 1+ edema at the ankle, with a 14-cm x 20-cm warm, erythematous area just above the ankle and extending proximally.

His labs found an HCT of 44 and a WBC of 12,000. What do you recommend?
 

A) Vascular duplex exam

B) 1st generation cephalosporin

C) 1st generation cephalosporin + TMP/Sulfa

D) Oral clindamycin

E) IV vancomycin



This patient has cellulitis and should receive a beta lactam antibiotic, which will have the best coverage and lowest minimal inhibitory concentration for the likely organism, beta hemolytic streptococci. Clindamycin would likely work, but it has greater side effects. This patient does not need coverage for methicillin-resistant staphylococcus aureus (MRSA). I know many of you, if not most, know this, but I want to go through relevant data and formal recommendations, because of a recent call I received from a patient.

My patient had a full body rash after receiving cephalexin + TMP/sulfa [trimethoprim-sulfamethoxazole] treatment for cellulitis. In recent years the addition of TMP/sulfa to strep treatment to also cover MRSA has become popular, especially in emergency department and urgent care settings.

Moran and colleagues studied cephalexin + TMP/sulfa vs. cephalexin and placebo in patients with uncomplicated cellulitis.1 The outcome measured was clinical cure, and there was no difference between groups; clinical cure occurred in 182 (83.5%) of 218 participants in the cephalexin plus TMP/sulfa group vs. 165 (85.5%) of 193 in the cephalexin group (difference, −2.0%; 95% confidence interval, −9.7% to 5.7%; P = .50).

Jeng and colleagues studied patients admitted for a cellulitis, and evaluated the patients’ response to beta-lactam antibiotics.2 Patients had acute and convalescent serologies for beta hemolytic strep. Almost all evaluable patients with positive strep studies (97%) responded to beta-lactams, and 21 of 23 (91%) with negative studies responded to beta-lactams (overall response rate 95%). This study was done during a time of high MRSA prevalence.

The most recent Infectious Diseases Society of America guidelines for skin and soft tissue infections, recommend oral penicillin, cephalexin, dicloxacillin, or clindamycin for mild cellulitis, and IV equivalent if patients have moderate cellulitis.3 If abscesses are present, then drainage is recommended and MRSA coverage. Kamath and colleagues reported on how closely guidelines for skin and soft tissue infections were followed.4 In patients with mild cellulitis, only 36% received guideline-suggested antibiotics. The most common antibiotic prescribed that was outside the guidelines was trimethoprim-sulfamethoxazole.
 

Myth: Cellulitis treatment should include MRSA coverage.

My advice: Stick with beta-lactam antibiotics, unless an abscess is present. There is no need to add MRSA coverage for initial treatment of mild to moderate cellulitis.

Dr. Paauw is professor of medicine in the division of general internal medicine at the University of Washington, Seattle, and he serves as third-year medical student clerkship director at the University of Washington. He is a member of the editorial advisory board of Internal Medicine News. Dr. Paauw has no conflicts to disclose. Contact him at imnews@mdedge.com.

References

1. Moran GJ et al. Effect of cephalexin plus trimethoprim-sulfamethoxazole vs. cephalexin alone on clinical cure of uncomplicated cellulitis: A randomized clinical trial. JAMA 2017 May 23;317(20):2088-96.

2. Jeng Arthur et al. The role of beta-hemolytic streptococci in causing diffuse, nonculturable cellulitis. Medicine. 2010;July;89(4):217-26.

3. Stevens DL et al. Practice guidelines for the diagnosis and management of skin and soft tissue infections: 2014 update by the Infectious Diseases Society of America. Clin Infect Dis. 2014;59(2):e10-e52.

4. Kamath RS et al. Guidelines vs. actual management of skin and soft tissue infections in the emergency department. Open Forum Infect Dis. 2018 Jan 12;5(1):ofx188.
 

Publications
Topics
Sections

An obese 64-year-old man with type 2 diabetes presents with redness and warmth of his lower left leg.

Paauw_Doug_SEATTLE_2019_web.jpg
Dr. Douglas S. Paauw

He noticed discomfort today and saw that his left lower leg had redness and was warm. He does not recall scratches or injury to his leg. He has not had fever or chills. He has no other symptoms. His diabetes has been well controlled with diet and metformin.

On exam, his blood pressure is 120/70, pulse is 80, temperature is 37 degrees Celsius.

In the left lower extremity, the patient had 1+ edema at the ankle, with a 14-cm x 20-cm warm, erythematous area just above the ankle and extending proximally.

His labs found an HCT of 44 and a WBC of 12,000. What do you recommend?
 

A) Vascular duplex exam

B) 1st generation cephalosporin

C) 1st generation cephalosporin + TMP/Sulfa

D) Oral clindamycin

E) IV vancomycin



This patient has cellulitis and should receive a beta lactam antibiotic, which will have the best coverage and lowest minimal inhibitory concentration for the likely organism, beta hemolytic streptococci. Clindamycin would likely work, but it has greater side effects. This patient does not need coverage for methicillin-resistant staphylococcus aureus (MRSA). I know many of you, if not most, know this, but I want to go through relevant data and formal recommendations, because of a recent call I received from a patient.

My patient had a full body rash after receiving cephalexin + TMP/sulfa [trimethoprim-sulfamethoxazole] treatment for cellulitis. In recent years the addition of TMP/sulfa to strep treatment to also cover MRSA has become popular, especially in emergency department and urgent care settings.

Moran and colleagues studied cephalexin + TMP/sulfa vs. cephalexin and placebo in patients with uncomplicated cellulitis.1 The outcome measured was clinical cure, and there was no difference between groups; clinical cure occurred in 182 (83.5%) of 218 participants in the cephalexin plus TMP/sulfa group vs. 165 (85.5%) of 193 in the cephalexin group (difference, −2.0%; 95% confidence interval, −9.7% to 5.7%; P = .50).

Jeng and colleagues studied patients admitted for a cellulitis, and evaluated the patients’ response to beta-lactam antibiotics.2 Patients had acute and convalescent serologies for beta hemolytic strep. Almost all evaluable patients with positive strep studies (97%) responded to beta-lactams, and 21 of 23 (91%) with negative studies responded to beta-lactams (overall response rate 95%). This study was done during a time of high MRSA prevalence.

The most recent Infectious Diseases Society of America guidelines for skin and soft tissue infections, recommend oral penicillin, cephalexin, dicloxacillin, or clindamycin for mild cellulitis, and IV equivalent if patients have moderate cellulitis.3 If abscesses are present, then drainage is recommended and MRSA coverage. Kamath and colleagues reported on how closely guidelines for skin and soft tissue infections were followed.4 In patients with mild cellulitis, only 36% received guideline-suggested antibiotics. The most common antibiotic prescribed that was outside the guidelines was trimethoprim-sulfamethoxazole.
 

Myth: Cellulitis treatment should include MRSA coverage.

My advice: Stick with beta-lactam antibiotics, unless an abscess is present. There is no need to add MRSA coverage for initial treatment of mild to moderate cellulitis.

Dr. Paauw is professor of medicine in the division of general internal medicine at the University of Washington, Seattle, and he serves as third-year medical student clerkship director at the University of Washington. He is a member of the editorial advisory board of Internal Medicine News. Dr. Paauw has no conflicts to disclose. Contact him at imnews@mdedge.com.

References

1. Moran GJ et al. Effect of cephalexin plus trimethoprim-sulfamethoxazole vs. cephalexin alone on clinical cure of uncomplicated cellulitis: A randomized clinical trial. JAMA 2017 May 23;317(20):2088-96.

2. Jeng Arthur et al. The role of beta-hemolytic streptococci in causing diffuse, nonculturable cellulitis. Medicine. 2010;July;89(4):217-26.

3. Stevens DL et al. Practice guidelines for the diagnosis and management of skin and soft tissue infections: 2014 update by the Infectious Diseases Society of America. Clin Infect Dis. 2014;59(2):e10-e52.

4. Kamath RS et al. Guidelines vs. actual management of skin and soft tissue infections in the emergency department. Open Forum Infect Dis. 2018 Jan 12;5(1):ofx188.
 

An obese 64-year-old man with type 2 diabetes presents with redness and warmth of his lower left leg.

Paauw_Doug_SEATTLE_2019_web.jpg
Dr. Douglas S. Paauw

He noticed discomfort today and saw that his left lower leg had redness and was warm. He does not recall scratches or injury to his leg. He has not had fever or chills. He has no other symptoms. His diabetes has been well controlled with diet and metformin.

On exam, his blood pressure is 120/70, pulse is 80, temperature is 37 degrees Celsius.

In the left lower extremity, the patient had 1+ edema at the ankle, with a 14-cm x 20-cm warm, erythematous area just above the ankle and extending proximally.

His labs found an HCT of 44 and a WBC of 12,000. What do you recommend?
 

A) Vascular duplex exam

B) 1st generation cephalosporin

C) 1st generation cephalosporin + TMP/Sulfa

D) Oral clindamycin

E) IV vancomycin



This patient has cellulitis and should receive a beta lactam antibiotic, which will have the best coverage and lowest minimal inhibitory concentration for the likely organism, beta hemolytic streptococci. Clindamycin would likely work, but it has greater side effects. This patient does not need coverage for methicillin-resistant staphylococcus aureus (MRSA). I know many of you, if not most, know this, but I want to go through relevant data and formal recommendations, because of a recent call I received from a patient.

My patient had a full body rash after receiving cephalexin + TMP/sulfa [trimethoprim-sulfamethoxazole] treatment for cellulitis. In recent years the addition of TMP/sulfa to strep treatment to also cover MRSA has become popular, especially in emergency department and urgent care settings.

Moran and colleagues studied cephalexin + TMP/sulfa vs. cephalexin and placebo in patients with uncomplicated cellulitis.1 The outcome measured was clinical cure, and there was no difference between groups; clinical cure occurred in 182 (83.5%) of 218 participants in the cephalexin plus TMP/sulfa group vs. 165 (85.5%) of 193 in the cephalexin group (difference, −2.0%; 95% confidence interval, −9.7% to 5.7%; P = .50).

Jeng and colleagues studied patients admitted for a cellulitis, and evaluated the patients’ response to beta-lactam antibiotics.2 Patients had acute and convalescent serologies for beta hemolytic strep. Almost all evaluable patients with positive strep studies (97%) responded to beta-lactams, and 21 of 23 (91%) with negative studies responded to beta-lactams (overall response rate 95%). This study was done during a time of high MRSA prevalence.

The most recent Infectious Diseases Society of America guidelines for skin and soft tissue infections, recommend oral penicillin, cephalexin, dicloxacillin, or clindamycin for mild cellulitis, and IV equivalent if patients have moderate cellulitis.3 If abscesses are present, then drainage is recommended and MRSA coverage. Kamath and colleagues reported on how closely guidelines for skin and soft tissue infections were followed.4 In patients with mild cellulitis, only 36% received guideline-suggested antibiotics. The most common antibiotic prescribed that was outside the guidelines was trimethoprim-sulfamethoxazole.
 

Myth: Cellulitis treatment should include MRSA coverage.

My advice: Stick with beta-lactam antibiotics, unless an abscess is present. There is no need to add MRSA coverage for initial treatment of mild to moderate cellulitis.

Dr. Paauw is professor of medicine in the division of general internal medicine at the University of Washington, Seattle, and he serves as third-year medical student clerkship director at the University of Washington. He is a member of the editorial advisory board of Internal Medicine News. Dr. Paauw has no conflicts to disclose. Contact him at imnews@mdedge.com.

References

1. Moran GJ et al. Effect of cephalexin plus trimethoprim-sulfamethoxazole vs. cephalexin alone on clinical cure of uncomplicated cellulitis: A randomized clinical trial. JAMA 2017 May 23;317(20):2088-96.

2. Jeng Arthur et al. The role of beta-hemolytic streptococci in causing diffuse, nonculturable cellulitis. Medicine. 2010;July;89(4):217-26.

3. Stevens DL et al. Practice guidelines for the diagnosis and management of skin and soft tissue infections: 2014 update by the Infectious Diseases Society of America. Clin Infect Dis. 2014;59(2):e10-e52.

4. Kamath RS et al. Guidelines vs. actual management of skin and soft tissue infections in the emergency department. Open Forum Infect Dis. 2018 Jan 12;5(1):ofx188.
 

Publications
Publications
Topics
Article Type
Sections
Disallow All Ads
Content Gating
No Gating (article Unlocked/Free)
Alternative CME
Disqus Comments
Default
Use ProPublica
Hide sidebar & use full width
render the right sidebar.
Conference Recap Checkbox
Not Conference Recap
Clinical Edge
Display the Slideshow in this Article
Medscape Article
Display survey writer

Colonoscopy prep suggestions for those who hate it

Article Type
Changed
Mon, 01/25/2021 - 15:32

 

A 61-year-old man is seen for a primary care visit. He has a history of colonic polyps (tubular adenoma) on two previous colonoscopies (at age 50 and 55). He has been on an appropriate 5-year schedule, but is overdue for his colonoscopy. He did not follow up with messages from his gastroenterologist for scheduling his colonoscopy last year. He explains he really hates the whole preparation for colonoscopy, but does realize he needs to follow up, and is willing to do so now. What do you recommend for colonoscopy prep?

Paauw_Doug_SEATTLE_2019_web2.jpg
Dr. Douglas S. Paauw



A) Diet as usual until 5 p.m. day before, then clear liquid diet. Start GoLYTELY (1 gallon) night before procedure.

B) Low-fiber diet X2 days, clear liquid diet day before procedure, GoLYTELY (1 gallon) night before procedure.

C) Low residue diet X3 days, SUPREP the night before the procedure.

D) Low residue diet X2 days, followed by clear liquid diet the day before the procedure, SUPREP the night before the procedure.
 

It is common for patients to be reluctant to follow recommendations for colonoscopy due to dreading the prep. I would recommend choice C here, as the least difficult bowel preparation for colonoscopy.

Gastroenterologists are usually the ones to recommend the bowel prep that they want their patients to follow. When patients avoid needed colonoscopies because they hate the prep they are advised to take, I think it is good for primary care professionals to step in and discuss evidence-based alternatives.

Major diet change for several days before colonoscopy is difficult for many patients. Standard advice is that patients eat only low-fiber foods starting 3 days before the procedure. Patients are advised to switch to a completely clear liquid diet 1-2 days before the colonoscopy.

Are there more tolerable diets to offer patients?

Soweid and colleagues randomized 200 patients to a low residue diet for the three meals the day before colonoscopy vs. clear liquid diet.1 The low residue diet allowed patients to eat meat, eggs, cheese, bread, rice, and ice cream. Not surprisingly, patients tolerated the low residue diet better with statistically significantly less nausea, vomiting, weakness, headache, sleep difficulties, and hunger. The patients in the low residue diet group also had better bowel prep than did those in the clear liquid diet group (81% vs. 52%, P less than 0.001).1

In a recent meta- analysis, low residue diets were comparable to clear liquid diets in regard to adequacy of bowel prep and for detection of polyps.2 Patients who followed low residue diets had statistically significantly less headaches, nausea, vomiting, and hunger. Very importantly, patients who followed low residue diets showed an increased willingness to repeat it, compared with those who followed a clear liquid diet (P less than .005; odds ratio, 2.23; 95% confidence interval, 1.28-3.89).2

What alternatives to GoLYTELY exist?

Another part of the bowel prep that patients struggle with is drinking a gallon of GoLYTELY (polyethylene glycol/electrolytes). Drinking that amount of this nasty stuff is never welcome.

 

 

There are a number of lower-volume alternatives that are as effective as GoLYTELY. Sarvepalli and colleagues did a retrospective study of 75,874 patients who had a colonoscopy in the Cleveland Clinic health system.3 The choice of bowel prep was not associated with adenoma detection.

Patients who lower volume preparations (2 quarts) SUPREP, MoviPrep, Osmoprep and HalfLytely had varying results of rates of inadequate bowel prep compared with patients who took GoLYTELY. Results for patients taking SUPREP and MoviPrep were statistically significantly better than for patients taking GoLYTELY. Results for patients taking OsmoPrep were not statistically different from those for patients taking GoLYTELY. Rates of inadequate bowel prep were statistically higher, meaning worse, for patients taking HalfLytely vs. patients taking GoLYTELY.3

Gu and colleagues did a prospective study of bowel prep outcomes from 4,339 colonoscopies, involving 75 different endoscopists.4 There was a wide range of bowel preps used, including low- and high-volume bowel preps. The low-volume preparations, SUPREP (P less than .001), MoviPrep (P less than .004) and MiraLAX with Gatorade (P less than .001), were superior to GoLYTELY for bowel cleansing. This was based on scoring via the Boston Bowel Preparation Scale. All were better tolerated than GoLYTELY.



Myth: All patients need a clear liquid diet and GoLYTELY for their bowel prep.
 

Dr. Paauw is professor of medicine in the division of general internal medicine at the University of Washington, Seattle, and he serves as third-year medical student clerkship director at the University of Washington. He is a member of the editorial advisory board of Internal Medicine News. Dr. Paauw has no conflicts to disclose. Contact him at imnews@mdedge.com.

References

1. Soweid AM et al. A randomized single-blind trial of standard diet versus fiber-free diet with polyethylene glycol electrolyte solution for colonoscopy preparation. Endoscopy 2010;42:633-8.

2. Zhang X et al. Low-[residue] diet versus clear-liquid diet for bowel preparation before colonoscopy: meta-analysis and trial sequential analysis of randomized controlled trials. Gastrointest Endosc. 2020 Sep;92(3):508-18.

3. Sarvepalli S et al. Comparative effectiveness of commercial bowel preparations in ambulatory patients presenting for screening or surveillance colonoscopy. Dig Dis Sci. 2020 Jul 20. doi: 10.1007/s10620-020-06492-z.

4. Gu P et al. Comparing the real-world effectiveness of competing colonoscopy preparations: results of a prospective trial. Am J Gastroenterol. 2019;114(2):305-14.

Publications
Topics
Sections

 

A 61-year-old man is seen for a primary care visit. He has a history of colonic polyps (tubular adenoma) on two previous colonoscopies (at age 50 and 55). He has been on an appropriate 5-year schedule, but is overdue for his colonoscopy. He did not follow up with messages from his gastroenterologist for scheduling his colonoscopy last year. He explains he really hates the whole preparation for colonoscopy, but does realize he needs to follow up, and is willing to do so now. What do you recommend for colonoscopy prep?

Paauw_Doug_SEATTLE_2019_web2.jpg
Dr. Douglas S. Paauw



A) Diet as usual until 5 p.m. day before, then clear liquid diet. Start GoLYTELY (1 gallon) night before procedure.

B) Low-fiber diet X2 days, clear liquid diet day before procedure, GoLYTELY (1 gallon) night before procedure.

C) Low residue diet X3 days, SUPREP the night before the procedure.

D) Low residue diet X2 days, followed by clear liquid diet the day before the procedure, SUPREP the night before the procedure.
 

It is common for patients to be reluctant to follow recommendations for colonoscopy due to dreading the prep. I would recommend choice C here, as the least difficult bowel preparation for colonoscopy.

Gastroenterologists are usually the ones to recommend the bowel prep that they want their patients to follow. When patients avoid needed colonoscopies because they hate the prep they are advised to take, I think it is good for primary care professionals to step in and discuss evidence-based alternatives.

Major diet change for several days before colonoscopy is difficult for many patients. Standard advice is that patients eat only low-fiber foods starting 3 days before the procedure. Patients are advised to switch to a completely clear liquid diet 1-2 days before the colonoscopy.

Are there more tolerable diets to offer patients?

Soweid and colleagues randomized 200 patients to a low residue diet for the three meals the day before colonoscopy vs. clear liquid diet.1 The low residue diet allowed patients to eat meat, eggs, cheese, bread, rice, and ice cream. Not surprisingly, patients tolerated the low residue diet better with statistically significantly less nausea, vomiting, weakness, headache, sleep difficulties, and hunger. The patients in the low residue diet group also had better bowel prep than did those in the clear liquid diet group (81% vs. 52%, P less than 0.001).1

In a recent meta- analysis, low residue diets were comparable to clear liquid diets in regard to adequacy of bowel prep and for detection of polyps.2 Patients who followed low residue diets had statistically significantly less headaches, nausea, vomiting, and hunger. Very importantly, patients who followed low residue diets showed an increased willingness to repeat it, compared with those who followed a clear liquid diet (P less than .005; odds ratio, 2.23; 95% confidence interval, 1.28-3.89).2

What alternatives to GoLYTELY exist?

Another part of the bowel prep that patients struggle with is drinking a gallon of GoLYTELY (polyethylene glycol/electrolytes). Drinking that amount of this nasty stuff is never welcome.

 

 

There are a number of lower-volume alternatives that are as effective as GoLYTELY. Sarvepalli and colleagues did a retrospective study of 75,874 patients who had a colonoscopy in the Cleveland Clinic health system.3 The choice of bowel prep was not associated with adenoma detection.

Patients who lower volume preparations (2 quarts) SUPREP, MoviPrep, Osmoprep and HalfLytely had varying results of rates of inadequate bowel prep compared with patients who took GoLYTELY. Results for patients taking SUPREP and MoviPrep were statistically significantly better than for patients taking GoLYTELY. Results for patients taking OsmoPrep were not statistically different from those for patients taking GoLYTELY. Rates of inadequate bowel prep were statistically higher, meaning worse, for patients taking HalfLytely vs. patients taking GoLYTELY.3

Gu and colleagues did a prospective study of bowel prep outcomes from 4,339 colonoscopies, involving 75 different endoscopists.4 There was a wide range of bowel preps used, including low- and high-volume bowel preps. The low-volume preparations, SUPREP (P less than .001), MoviPrep (P less than .004) and MiraLAX with Gatorade (P less than .001), were superior to GoLYTELY for bowel cleansing. This was based on scoring via the Boston Bowel Preparation Scale. All were better tolerated than GoLYTELY.



Myth: All patients need a clear liquid diet and GoLYTELY for their bowel prep.
 

Dr. Paauw is professor of medicine in the division of general internal medicine at the University of Washington, Seattle, and he serves as third-year medical student clerkship director at the University of Washington. He is a member of the editorial advisory board of Internal Medicine News. Dr. Paauw has no conflicts to disclose. Contact him at imnews@mdedge.com.

References

1. Soweid AM et al. A randomized single-blind trial of standard diet versus fiber-free diet with polyethylene glycol electrolyte solution for colonoscopy preparation. Endoscopy 2010;42:633-8.

2. Zhang X et al. Low-[residue] diet versus clear-liquid diet for bowel preparation before colonoscopy: meta-analysis and trial sequential analysis of randomized controlled trials. Gastrointest Endosc. 2020 Sep;92(3):508-18.

3. Sarvepalli S et al. Comparative effectiveness of commercial bowel preparations in ambulatory patients presenting for screening or surveillance colonoscopy. Dig Dis Sci. 2020 Jul 20. doi: 10.1007/s10620-020-06492-z.

4. Gu P et al. Comparing the real-world effectiveness of competing colonoscopy preparations: results of a prospective trial. Am J Gastroenterol. 2019;114(2):305-14.

 

A 61-year-old man is seen for a primary care visit. He has a history of colonic polyps (tubular adenoma) on two previous colonoscopies (at age 50 and 55). He has been on an appropriate 5-year schedule, but is overdue for his colonoscopy. He did not follow up with messages from his gastroenterologist for scheduling his colonoscopy last year. He explains he really hates the whole preparation for colonoscopy, but does realize he needs to follow up, and is willing to do so now. What do you recommend for colonoscopy prep?

Paauw_Doug_SEATTLE_2019_web2.jpg
Dr. Douglas S. Paauw



A) Diet as usual until 5 p.m. day before, then clear liquid diet. Start GoLYTELY (1 gallon) night before procedure.

B) Low-fiber diet X2 days, clear liquid diet day before procedure, GoLYTELY (1 gallon) night before procedure.

C) Low residue diet X3 days, SUPREP the night before the procedure.

D) Low residue diet X2 days, followed by clear liquid diet the day before the procedure, SUPREP the night before the procedure.
 

It is common for patients to be reluctant to follow recommendations for colonoscopy due to dreading the prep. I would recommend choice C here, as the least difficult bowel preparation for colonoscopy.

Gastroenterologists are usually the ones to recommend the bowel prep that they want their patients to follow. When patients avoid needed colonoscopies because they hate the prep they are advised to take, I think it is good for primary care professionals to step in and discuss evidence-based alternatives.

Major diet change for several days before colonoscopy is difficult for many patients. Standard advice is that patients eat only low-fiber foods starting 3 days before the procedure. Patients are advised to switch to a completely clear liquid diet 1-2 days before the colonoscopy.

Are there more tolerable diets to offer patients?

Soweid and colleagues randomized 200 patients to a low residue diet for the three meals the day before colonoscopy vs. clear liquid diet.1 The low residue diet allowed patients to eat meat, eggs, cheese, bread, rice, and ice cream. Not surprisingly, patients tolerated the low residue diet better with statistically significantly less nausea, vomiting, weakness, headache, sleep difficulties, and hunger. The patients in the low residue diet group also had better bowel prep than did those in the clear liquid diet group (81% vs. 52%, P less than 0.001).1

In a recent meta- analysis, low residue diets were comparable to clear liquid diets in regard to adequacy of bowel prep and for detection of polyps.2 Patients who followed low residue diets had statistically significantly less headaches, nausea, vomiting, and hunger. Very importantly, patients who followed low residue diets showed an increased willingness to repeat it, compared with those who followed a clear liquid diet (P less than .005; odds ratio, 2.23; 95% confidence interval, 1.28-3.89).2

What alternatives to GoLYTELY exist?

Another part of the bowel prep that patients struggle with is drinking a gallon of GoLYTELY (polyethylene glycol/electrolytes). Drinking that amount of this nasty stuff is never welcome.

 

 

There are a number of lower-volume alternatives that are as effective as GoLYTELY. Sarvepalli and colleagues did a retrospective study of 75,874 patients who had a colonoscopy in the Cleveland Clinic health system.3 The choice of bowel prep was not associated with adenoma detection.

Patients who lower volume preparations (2 quarts) SUPREP, MoviPrep, Osmoprep and HalfLytely had varying results of rates of inadequate bowel prep compared with patients who took GoLYTELY. Results for patients taking SUPREP and MoviPrep were statistically significantly better than for patients taking GoLYTELY. Results for patients taking OsmoPrep were not statistically different from those for patients taking GoLYTELY. Rates of inadequate bowel prep were statistically higher, meaning worse, for patients taking HalfLytely vs. patients taking GoLYTELY.3

Gu and colleagues did a prospective study of bowel prep outcomes from 4,339 colonoscopies, involving 75 different endoscopists.4 There was a wide range of bowel preps used, including low- and high-volume bowel preps. The low-volume preparations, SUPREP (P less than .001), MoviPrep (P less than .004) and MiraLAX with Gatorade (P less than .001), were superior to GoLYTELY for bowel cleansing. This was based on scoring via the Boston Bowel Preparation Scale. All were better tolerated than GoLYTELY.



Myth: All patients need a clear liquid diet and GoLYTELY for their bowel prep.
 

Dr. Paauw is professor of medicine in the division of general internal medicine at the University of Washington, Seattle, and he serves as third-year medical student clerkship director at the University of Washington. He is a member of the editorial advisory board of Internal Medicine News. Dr. Paauw has no conflicts to disclose. Contact him at imnews@mdedge.com.

References

1. Soweid AM et al. A randomized single-blind trial of standard diet versus fiber-free diet with polyethylene glycol electrolyte solution for colonoscopy preparation. Endoscopy 2010;42:633-8.

2. Zhang X et al. Low-[residue] diet versus clear-liquid diet for bowel preparation before colonoscopy: meta-analysis and trial sequential analysis of randomized controlled trials. Gastrointest Endosc. 2020 Sep;92(3):508-18.

3. Sarvepalli S et al. Comparative effectiveness of commercial bowel preparations in ambulatory patients presenting for screening or surveillance colonoscopy. Dig Dis Sci. 2020 Jul 20. doi: 10.1007/s10620-020-06492-z.

4. Gu P et al. Comparing the real-world effectiveness of competing colonoscopy preparations: results of a prospective trial. Am J Gastroenterol. 2019;114(2):305-14.

Publications
Publications
Topics
Article Type
Sections
Disallow All Ads
Content Gating
No Gating (article Unlocked/Free)
Alternative CME
Disqus Comments
Default
Use ProPublica
Hide sidebar & use full width
render the right sidebar.
Conference Recap Checkbox
Not Conference Recap
Clinical Edge
Display the Slideshow in this Article
Medscape Article

Patient with CKD: Contrast or no contrast?

Article Type
Changed
Wed, 12/09/2020 - 10:39

A 67-year-old man with stage 3 chronic kidney disease (CKD) develops abdominal pain over 24 hours. He has had low grade fevers and nausea. He has a history of colon cancer and had a resection four years ago. Abdominal exam reveals tenderness to palpation, including rebound tenderness in his right lower quadrant. Labs: hemoglobin: 13; hematocrit: 39; white blood cells: 18,000; platelets: 333; blood urea nitrogen: 28; creatinine: 1.8 (estimated glomerular filtration rate: 37); sodium: 136; potassium: 3.9; bicarbonate: 24; chlorine: 105; and lipase: 10.

Paauw_Doug_SEATTLE_2019_web.jpg
Dr. Douglas S. Paauw

What testing would you recommend?

A) Ultrasound

B) Non contrast computed tomography (CT)

C) Contrast CT

D) MRI without gadolinium

The correct answer here is to get a contrast CT scan, as it will give you the most appropriate diagnostic information.

For years, we have hesitated to order contrast studies in our patients with CKD, for fear of causing contrast-induced nephrotoxicity. We might choose less helpful studies that avoid contrast, or might not obtain imaging that is needed. Over the years I have especially seen this in the case of avoiding computed tomography angiography (CTA) for evaluation of pulmonary embolus and choosing the much less useful ventilation/perfusion scan. The problem arises with the fact that patients with CKD are more likely to develop worsening renal function when they get sick.

The assumption had been that when kidney injury occurred after contrast that it was due to the contrast. Many recent studies refute this assumption. Lee and colleagues performed an analysis of six retrospective studies involving a total of 55,963 participants. They found that patients with CKD receiving contrast material did not have an increased risk of deteriorating renal function compared with those without CKD (odds ratio, 1.07; 95% confidence interval, 0.98-1.17).1

The early studies reporting contrast-induced renal disease were in patients who received high osmolality contrast agents.2 Most patients now receive low osmolality agents, with less nephrotoxicity.3
 

Key points of guidelines

This year, the American College of Radiology and the National Kidney Foundation put out joint guidelines that helped clarify why there is a diminished concern for contrast-induced kidney disease in the modern era.4 Below are some of the key points of these guidelines:

  • The risk of contrast-induced acute kidney injury (AKI) from intravenous iodinated contrast media is lower than previously thought.
  • Necessary contrast material–enhanced CT without a suitable alternative should not be avoided solely on the basis of contrast-induced chronic kidney insufficiency risk.
  • Contrast-induced AKI risk should be determined primarily by using CKD stage and AKI.
  • Patients at high risk for contrast-induced kidney injury include those with recent AKI and those with estimated glomerular filtration rate (eGFR) less than 30 mL/min per 1.73 m2.

Data supporting guidelines

The data from several studies used to support these recommendations were impressive, showing just how low the risk for contrast-induced AKI is in most patients. In these studies, the risk of contrast-induced AKI has been estimated to be near 0% for patients with an eGFR greater than or equal to 45 and 0%-2% for patients with an eGFR of 30-44.5-7 This information and recommendations make imaging much easier. In most of our patients, we can get contrast studies when we need them. The group to be concerned about are patients with eGFRs less than 30. The guidelines single out this group as the patients where risk/benefit needs to be calculated before proceeding with the study, and to use prophylactic saline hydration in patients not undergoing dialysis.


Myth: Contrast-induced renal disease is common.
 

Dr. Paauw is professor of medicine in the division of general internal medicine at the University of Washington, Seattle, and he serves as third-year medical student clerkship director at the University of Washington. He is a member of the editorial advisory board of Internal Medicine News. Dr. Paauw has no conflicts to disclose. Contact him at imnews@mdedge.com.

References

1. Lee YC et al. Contrast-induced acute kidney injury among patients with chronic kidney disease undergoing imaging studies: A meta-analysis. Am J Roentgenol. 2019 Oct;213(4):728-35.

2. Luk L et al. Intravenous contrast-induced nephropathy: The rise and fall of a threatening idea. Adv Chronic Kidney Dis. 2017 May;24(3):169-75.

3. Goldfarb S et al. Low-osmolality contrast media and the risk of contrast-associated nephrotoxicity. Invest Radiol. 1993;28(Suppl 5):7-10.

4. Davenport MS, et al. Use of intravenous iodinated contrast media in patients with kidney disease: Consensus statements from the American College of Radiology and the National Kidney Foundation. Kidney Med. 2020 Jan 22;2(1):85-93.

5. Davenport MS et al. Contrast material–induced nephrotoxicity and intravenous low-osmolality iodinated contrast material. Radiology. 2013;267(1):94-105.

6. McDonald RJ et al. Intravenous contrast material–induced nephropathy: Causal or coincident phenomenon? Radiology. 2013;267(1):106-18.

7. McDonald JS et al. Risk of intravenous contrast material–mediated acute kidney injury: A propensity scorematched study stratified by baseline-estimated glomerular filtration rate. Radiology. 2014;271(1):65-73.

Publications
Topics
Sections

A 67-year-old man with stage 3 chronic kidney disease (CKD) develops abdominal pain over 24 hours. He has had low grade fevers and nausea. He has a history of colon cancer and had a resection four years ago. Abdominal exam reveals tenderness to palpation, including rebound tenderness in his right lower quadrant. Labs: hemoglobin: 13; hematocrit: 39; white blood cells: 18,000; platelets: 333; blood urea nitrogen: 28; creatinine: 1.8 (estimated glomerular filtration rate: 37); sodium: 136; potassium: 3.9; bicarbonate: 24; chlorine: 105; and lipase: 10.

Paauw_Doug_SEATTLE_2019_web.jpg
Dr. Douglas S. Paauw

What testing would you recommend?

A) Ultrasound

B) Non contrast computed tomography (CT)

C) Contrast CT

D) MRI without gadolinium

The correct answer here is to get a contrast CT scan, as it will give you the most appropriate diagnostic information.

For years, we have hesitated to order contrast studies in our patients with CKD, for fear of causing contrast-induced nephrotoxicity. We might choose less helpful studies that avoid contrast, or might not obtain imaging that is needed. Over the years I have especially seen this in the case of avoiding computed tomography angiography (CTA) for evaluation of pulmonary embolus and choosing the much less useful ventilation/perfusion scan. The problem arises with the fact that patients with CKD are more likely to develop worsening renal function when they get sick.

The assumption had been that when kidney injury occurred after contrast that it was due to the contrast. Many recent studies refute this assumption. Lee and colleagues performed an analysis of six retrospective studies involving a total of 55,963 participants. They found that patients with CKD receiving contrast material did not have an increased risk of deteriorating renal function compared with those without CKD (odds ratio, 1.07; 95% confidence interval, 0.98-1.17).1

The early studies reporting contrast-induced renal disease were in patients who received high osmolality contrast agents.2 Most patients now receive low osmolality agents, with less nephrotoxicity.3
 

Key points of guidelines

This year, the American College of Radiology and the National Kidney Foundation put out joint guidelines that helped clarify why there is a diminished concern for contrast-induced kidney disease in the modern era.4 Below are some of the key points of these guidelines:

  • The risk of contrast-induced acute kidney injury (AKI) from intravenous iodinated contrast media is lower than previously thought.
  • Necessary contrast material–enhanced CT without a suitable alternative should not be avoided solely on the basis of contrast-induced chronic kidney insufficiency risk.
  • Contrast-induced AKI risk should be determined primarily by using CKD stage and AKI.
  • Patients at high risk for contrast-induced kidney injury include those with recent AKI and those with estimated glomerular filtration rate (eGFR) less than 30 mL/min per 1.73 m2.

Data supporting guidelines

The data from several studies used to support these recommendations were impressive, showing just how low the risk for contrast-induced AKI is in most patients. In these studies, the risk of contrast-induced AKI has been estimated to be near 0% for patients with an eGFR greater than or equal to 45 and 0%-2% for patients with an eGFR of 30-44.5-7 This information and recommendations make imaging much easier. In most of our patients, we can get contrast studies when we need them. The group to be concerned about are patients with eGFRs less than 30. The guidelines single out this group as the patients where risk/benefit needs to be calculated before proceeding with the study, and to use prophylactic saline hydration in patients not undergoing dialysis.


Myth: Contrast-induced renal disease is common.
 

Dr. Paauw is professor of medicine in the division of general internal medicine at the University of Washington, Seattle, and he serves as third-year medical student clerkship director at the University of Washington. He is a member of the editorial advisory board of Internal Medicine News. Dr. Paauw has no conflicts to disclose. Contact him at imnews@mdedge.com.

References

1. Lee YC et al. Contrast-induced acute kidney injury among patients with chronic kidney disease undergoing imaging studies: A meta-analysis. Am J Roentgenol. 2019 Oct;213(4):728-35.

2. Luk L et al. Intravenous contrast-induced nephropathy: The rise and fall of a threatening idea. Adv Chronic Kidney Dis. 2017 May;24(3):169-75.

3. Goldfarb S et al. Low-osmolality contrast media and the risk of contrast-associated nephrotoxicity. Invest Radiol. 1993;28(Suppl 5):7-10.

4. Davenport MS, et al. Use of intravenous iodinated contrast media in patients with kidney disease: Consensus statements from the American College of Radiology and the National Kidney Foundation. Kidney Med. 2020 Jan 22;2(1):85-93.

5. Davenport MS et al. Contrast material–induced nephrotoxicity and intravenous low-osmolality iodinated contrast material. Radiology. 2013;267(1):94-105.

6. McDonald RJ et al. Intravenous contrast material–induced nephropathy: Causal or coincident phenomenon? Radiology. 2013;267(1):106-18.

7. McDonald JS et al. Risk of intravenous contrast material–mediated acute kidney injury: A propensity scorematched study stratified by baseline-estimated glomerular filtration rate. Radiology. 2014;271(1):65-73.

A 67-year-old man with stage 3 chronic kidney disease (CKD) develops abdominal pain over 24 hours. He has had low grade fevers and nausea. He has a history of colon cancer and had a resection four years ago. Abdominal exam reveals tenderness to palpation, including rebound tenderness in his right lower quadrant. Labs: hemoglobin: 13; hematocrit: 39; white blood cells: 18,000; platelets: 333; blood urea nitrogen: 28; creatinine: 1.8 (estimated glomerular filtration rate: 37); sodium: 136; potassium: 3.9; bicarbonate: 24; chlorine: 105; and lipase: 10.

Paauw_Doug_SEATTLE_2019_web.jpg
Dr. Douglas S. Paauw

What testing would you recommend?

A) Ultrasound

B) Non contrast computed tomography (CT)

C) Contrast CT

D) MRI without gadolinium

The correct answer here is to get a contrast CT scan, as it will give you the most appropriate diagnostic information.

For years, we have hesitated to order contrast studies in our patients with CKD, for fear of causing contrast-induced nephrotoxicity. We might choose less helpful studies that avoid contrast, or might not obtain imaging that is needed. Over the years I have especially seen this in the case of avoiding computed tomography angiography (CTA) for evaluation of pulmonary embolus and choosing the much less useful ventilation/perfusion scan. The problem arises with the fact that patients with CKD are more likely to develop worsening renal function when they get sick.

The assumption had been that when kidney injury occurred after contrast that it was due to the contrast. Many recent studies refute this assumption. Lee and colleagues performed an analysis of six retrospective studies involving a total of 55,963 participants. They found that patients with CKD receiving contrast material did not have an increased risk of deteriorating renal function compared with those without CKD (odds ratio, 1.07; 95% confidence interval, 0.98-1.17).1

The early studies reporting contrast-induced renal disease were in patients who received high osmolality contrast agents.2 Most patients now receive low osmolality agents, with less nephrotoxicity.3
 

Key points of guidelines

This year, the American College of Radiology and the National Kidney Foundation put out joint guidelines that helped clarify why there is a diminished concern for contrast-induced kidney disease in the modern era.4 Below are some of the key points of these guidelines:

  • The risk of contrast-induced acute kidney injury (AKI) from intravenous iodinated contrast media is lower than previously thought.
  • Necessary contrast material–enhanced CT without a suitable alternative should not be avoided solely on the basis of contrast-induced chronic kidney insufficiency risk.
  • Contrast-induced AKI risk should be determined primarily by using CKD stage and AKI.
  • Patients at high risk for contrast-induced kidney injury include those with recent AKI and those with estimated glomerular filtration rate (eGFR) less than 30 mL/min per 1.73 m2.

Data supporting guidelines

The data from several studies used to support these recommendations were impressive, showing just how low the risk for contrast-induced AKI is in most patients. In these studies, the risk of contrast-induced AKI has been estimated to be near 0% for patients with an eGFR greater than or equal to 45 and 0%-2% for patients with an eGFR of 30-44.5-7 This information and recommendations make imaging much easier. In most of our patients, we can get contrast studies when we need them. The group to be concerned about are patients with eGFRs less than 30. The guidelines single out this group as the patients where risk/benefit needs to be calculated before proceeding with the study, and to use prophylactic saline hydration in patients not undergoing dialysis.


Myth: Contrast-induced renal disease is common.
 

Dr. Paauw is professor of medicine in the division of general internal medicine at the University of Washington, Seattle, and he serves as third-year medical student clerkship director at the University of Washington. He is a member of the editorial advisory board of Internal Medicine News. Dr. Paauw has no conflicts to disclose. Contact him at imnews@mdedge.com.

References

1. Lee YC et al. Contrast-induced acute kidney injury among patients with chronic kidney disease undergoing imaging studies: A meta-analysis. Am J Roentgenol. 2019 Oct;213(4):728-35.

2. Luk L et al. Intravenous contrast-induced nephropathy: The rise and fall of a threatening idea. Adv Chronic Kidney Dis. 2017 May;24(3):169-75.

3. Goldfarb S et al. Low-osmolality contrast media and the risk of contrast-associated nephrotoxicity. Invest Radiol. 1993;28(Suppl 5):7-10.

4. Davenport MS, et al. Use of intravenous iodinated contrast media in patients with kidney disease: Consensus statements from the American College of Radiology and the National Kidney Foundation. Kidney Med. 2020 Jan 22;2(1):85-93.

5. Davenport MS et al. Contrast material–induced nephrotoxicity and intravenous low-osmolality iodinated contrast material. Radiology. 2013;267(1):94-105.

6. McDonald RJ et al. Intravenous contrast material–induced nephropathy: Causal or coincident phenomenon? Radiology. 2013;267(1):106-18.

7. McDonald JS et al. Risk of intravenous contrast material–mediated acute kidney injury: A propensity scorematched study stratified by baseline-estimated glomerular filtration rate. Radiology. 2014;271(1):65-73.

Publications
Publications
Topics
Article Type
Sections
Disallow All Ads
Content Gating
No Gating (article Unlocked/Free)
Alternative CME
Disqus Comments
Default
Use ProPublica
Hide sidebar & use full width
render the right sidebar.
Conference Recap Checkbox
Not Conference Recap
Clinical Edge
Display the Slideshow in this Article
Medscape Article

Is anemia due to folate deficiency a myth?

Article Type
Changed
Fri, 05/22/2020 - 12:39

A 46-year-old man who lives in Tacoma, Wash., is seen for fatigue. He has a no significant past medical history. He is not taking any medications. His physical exam is unremarkable. His hemoglobin is 12 gm/dL, hematocrit is 37 gm/dL, mean corpuscular volume (MCV) is 103 fL, and thyroid-stimulating hormone level is 1.2 mU/L.

What workup do you recommend?

A) B12, folate testing

B) Alcohol history, B12, folate testing

C) Alcohol history, B12 testing

I would choose doing a careful alcohol history and vitamin B12 testing.

Paauw_Doug_SEATTLE_2019_web.jpg
Dr. Douglas S. Paauw

Dr. Seppä and colleagues looked at all outpatients who had a blood count done over an 8-month period.1 A total of 9,527 blood counts were ordered, and 287 (3%) had macrocytosis.1 Further workup was done for 113 of the patients. The most common cause found for macrocytosis was alcohol abuse, in 74 (65%) of the patients (80% of the men and 36% of the women). In several studies, vitamin B12 deficiency was the cause of macrocytosis in 5%-7% of patients.2,3

In 1978, a study by Davidson and Hamilton looked at 200 consecutive patients with MCVs over 100, and were able to find a cause in 80%.4 Sixteen of these patients had a low B12 level and 10 had a low folate level.

In recent years, folate has become an extremely unlikely cause of macrocytic anemias. In 1998, the Food and Drug Administration required folic acid fortification of enriched grain products in the United States to help decrease the risk of neural tube defects. Similar fortification efforts were undertaken in Canada. Since 1998, anemia due to folate deficiency has essentially disappeared in individuals who have access to fortified grain products.

Joelson and colleagues looked at data on folate testing from the year prior to fortification of the grain supply (1997) and after (2004).5 They found that, in 1997, 4.8% of tests had a folate level less than 160 ng/mL compared with only 0.6% of tests in 2004.

When a more stringent cutoff for deficiency was used (94 ng/mL) 0.98% of tests were below that level in 1997, and 0.09% in 2004. The mean RBC folate level in 1997 was 420 ng/mL and rose to 697 ng/mL in 2004. Of the patients who did have low folate levels, only a minority had elevated MCVs.

Shojania et al. looked at folate testing in Canada after widespread fortification had started.6 They found that 0.5% of 2,154 serum folate levels were low and 0.7% of 560 red blood cell folate levels were low. Folate deficiency was not the cause of anemia in any of the patients with low folate levels.

Theisen-Toupal and colleagues did a retrospective study looking at folate testing over an 11-year period after fortification.7 The researchers examined the results of 84,187 assessments of folate levels. Forty-seven (0.056%) of the tests found patients with folate deficiency, 166 (0.197%), found patients with low-normal folate levels, 57,411 (68.195%) of tests yielded normal results, and 26,563 (31.552%) of tests found high folate levels. The opinion of the authors was that folate testing should be severely reduced or eliminated. Furthermore, the American Society for Clinical Pathology, as part of the Choosing Wisely campaign, states: “Do not order red blood cell folate levels at all.”8

So what does this all mean? We have been taught to have a reflex response to the evaluation of macrocytosis to test for B12 and folate. Neither of these are particularly common causes of macrocytosis, and in countries where there is grain fortification, folate deficiency is exceedingly uncommon, and should not be tested for early in any diagnostic process.
 

Dr. Paauw is professor of medicine in the division of general internal medicine at the University of Washington, Seattle, and he serves as third-year medical student clerkship director at the University of Washington. He is a member of the editorial advisory board of Internal Medicine News. Dr. Paauw has no conflicts to disclose. Contact him at imnews@mdedge.com.

References

1. Seppä K et al. Evaluation of macrocytosis by general practitioners. J Stud Alcohol. 1996 Jan;57(1):97-100.

2. Seppä K et al. Blood count and hematologic morphology in nonanemic macrocytosis: Differences between alcohol abuse and pernicious anemia. Alcohol. 1993 Sep-Oct;10(5):343-7.

3. Wymer A, Becker DM. Recognition and evaluation of red blood cell macrocytosis in the primary care setting. J Gen Intern Med. 1990 May-Jun;5(3):192-7.

4. Davidson RJ, Hamilton PJ. High mean red cell volume: Its incidence and significance in routine haematology. J Clin Pathol. 1978 May;31[5]:493-8.

5. Joelson DW, Fiebig EW. Diminished need for folate measurements among indigent populations in the post folic acid supplementation era. Arch Pathol Lab Med. 2007 Mar;131(3):477-80.

6. Shojania AM, von Kuster K. Ordering folate assays is no longer justified for investigation of anemias, in folic acid fortified countries. BMC Res Notes. 2010 Jan 25;3:22. doi: 10.1186/1756-0500-3-22.

7. Theisen-Toupal et al. Low yield of outpatient serum folate testing. JAMA Intern Med. 2014 Oct. doi: 10.1001/jamainternmed.2014.3593.

8. Choosing Wisely: American Society for Clinical Pathology, Oct. 19, 2017. Recommendation.

Publications
Topics
Sections

A 46-year-old man who lives in Tacoma, Wash., is seen for fatigue. He has a no significant past medical history. He is not taking any medications. His physical exam is unremarkable. His hemoglobin is 12 gm/dL, hematocrit is 37 gm/dL, mean corpuscular volume (MCV) is 103 fL, and thyroid-stimulating hormone level is 1.2 mU/L.

What workup do you recommend?

A) B12, folate testing

B) Alcohol history, B12, folate testing

C) Alcohol history, B12 testing

I would choose doing a careful alcohol history and vitamin B12 testing.

Paauw_Doug_SEATTLE_2019_web.jpg
Dr. Douglas S. Paauw

Dr. Seppä and colleagues looked at all outpatients who had a blood count done over an 8-month period.1 A total of 9,527 blood counts were ordered, and 287 (3%) had macrocytosis.1 Further workup was done for 113 of the patients. The most common cause found for macrocytosis was alcohol abuse, in 74 (65%) of the patients (80% of the men and 36% of the women). In several studies, vitamin B12 deficiency was the cause of macrocytosis in 5%-7% of patients.2,3

In 1978, a study by Davidson and Hamilton looked at 200 consecutive patients with MCVs over 100, and were able to find a cause in 80%.4 Sixteen of these patients had a low B12 level and 10 had a low folate level.

In recent years, folate has become an extremely unlikely cause of macrocytic anemias. In 1998, the Food and Drug Administration required folic acid fortification of enriched grain products in the United States to help decrease the risk of neural tube defects. Similar fortification efforts were undertaken in Canada. Since 1998, anemia due to folate deficiency has essentially disappeared in individuals who have access to fortified grain products.

Joelson and colleagues looked at data on folate testing from the year prior to fortification of the grain supply (1997) and after (2004).5 They found that, in 1997, 4.8% of tests had a folate level less than 160 ng/mL compared with only 0.6% of tests in 2004.

When a more stringent cutoff for deficiency was used (94 ng/mL) 0.98% of tests were below that level in 1997, and 0.09% in 2004. The mean RBC folate level in 1997 was 420 ng/mL and rose to 697 ng/mL in 2004. Of the patients who did have low folate levels, only a minority had elevated MCVs.

Shojania et al. looked at folate testing in Canada after widespread fortification had started.6 They found that 0.5% of 2,154 serum folate levels were low and 0.7% of 560 red blood cell folate levels were low. Folate deficiency was not the cause of anemia in any of the patients with low folate levels.

Theisen-Toupal and colleagues did a retrospective study looking at folate testing over an 11-year period after fortification.7 The researchers examined the results of 84,187 assessments of folate levels. Forty-seven (0.056%) of the tests found patients with folate deficiency, 166 (0.197%), found patients with low-normal folate levels, 57,411 (68.195%) of tests yielded normal results, and 26,563 (31.552%) of tests found high folate levels. The opinion of the authors was that folate testing should be severely reduced or eliminated. Furthermore, the American Society for Clinical Pathology, as part of the Choosing Wisely campaign, states: “Do not order red blood cell folate levels at all.”8

So what does this all mean? We have been taught to have a reflex response to the evaluation of macrocytosis to test for B12 and folate. Neither of these are particularly common causes of macrocytosis, and in countries where there is grain fortification, folate deficiency is exceedingly uncommon, and should not be tested for early in any diagnostic process.
 

Dr. Paauw is professor of medicine in the division of general internal medicine at the University of Washington, Seattle, and he serves as third-year medical student clerkship director at the University of Washington. He is a member of the editorial advisory board of Internal Medicine News. Dr. Paauw has no conflicts to disclose. Contact him at imnews@mdedge.com.

References

1. Seppä K et al. Evaluation of macrocytosis by general practitioners. J Stud Alcohol. 1996 Jan;57(1):97-100.

2. Seppä K et al. Blood count and hematologic morphology in nonanemic macrocytosis: Differences between alcohol abuse and pernicious anemia. Alcohol. 1993 Sep-Oct;10(5):343-7.

3. Wymer A, Becker DM. Recognition and evaluation of red blood cell macrocytosis in the primary care setting. J Gen Intern Med. 1990 May-Jun;5(3):192-7.

4. Davidson RJ, Hamilton PJ. High mean red cell volume: Its incidence and significance in routine haematology. J Clin Pathol. 1978 May;31[5]:493-8.

5. Joelson DW, Fiebig EW. Diminished need for folate measurements among indigent populations in the post folic acid supplementation era. Arch Pathol Lab Med. 2007 Mar;131(3):477-80.

6. Shojania AM, von Kuster K. Ordering folate assays is no longer justified for investigation of anemias, in folic acid fortified countries. BMC Res Notes. 2010 Jan 25;3:22. doi: 10.1186/1756-0500-3-22.

7. Theisen-Toupal et al. Low yield of outpatient serum folate testing. JAMA Intern Med. 2014 Oct. doi: 10.1001/jamainternmed.2014.3593.

8. Choosing Wisely: American Society for Clinical Pathology, Oct. 19, 2017. Recommendation.

A 46-year-old man who lives in Tacoma, Wash., is seen for fatigue. He has a no significant past medical history. He is not taking any medications. His physical exam is unremarkable. His hemoglobin is 12 gm/dL, hematocrit is 37 gm/dL, mean corpuscular volume (MCV) is 103 fL, and thyroid-stimulating hormone level is 1.2 mU/L.

What workup do you recommend?

A) B12, folate testing

B) Alcohol history, B12, folate testing

C) Alcohol history, B12 testing

I would choose doing a careful alcohol history and vitamin B12 testing.

Paauw_Doug_SEATTLE_2019_web.jpg
Dr. Douglas S. Paauw

Dr. Seppä and colleagues looked at all outpatients who had a blood count done over an 8-month period.1 A total of 9,527 blood counts were ordered, and 287 (3%) had macrocytosis.1 Further workup was done for 113 of the patients. The most common cause found for macrocytosis was alcohol abuse, in 74 (65%) of the patients (80% of the men and 36% of the women). In several studies, vitamin B12 deficiency was the cause of macrocytosis in 5%-7% of patients.2,3

In 1978, a study by Davidson and Hamilton looked at 200 consecutive patients with MCVs over 100, and were able to find a cause in 80%.4 Sixteen of these patients had a low B12 level and 10 had a low folate level.

In recent years, folate has become an extremely unlikely cause of macrocytic anemias. In 1998, the Food and Drug Administration required folic acid fortification of enriched grain products in the United States to help decrease the risk of neural tube defects. Similar fortification efforts were undertaken in Canada. Since 1998, anemia due to folate deficiency has essentially disappeared in individuals who have access to fortified grain products.

Joelson and colleagues looked at data on folate testing from the year prior to fortification of the grain supply (1997) and after (2004).5 They found that, in 1997, 4.8% of tests had a folate level less than 160 ng/mL compared with only 0.6% of tests in 2004.

When a more stringent cutoff for deficiency was used (94 ng/mL) 0.98% of tests were below that level in 1997, and 0.09% in 2004. The mean RBC folate level in 1997 was 420 ng/mL and rose to 697 ng/mL in 2004. Of the patients who did have low folate levels, only a minority had elevated MCVs.

Shojania et al. looked at folate testing in Canada after widespread fortification had started.6 They found that 0.5% of 2,154 serum folate levels were low and 0.7% of 560 red blood cell folate levels were low. Folate deficiency was not the cause of anemia in any of the patients with low folate levels.

Theisen-Toupal and colleagues did a retrospective study looking at folate testing over an 11-year period after fortification.7 The researchers examined the results of 84,187 assessments of folate levels. Forty-seven (0.056%) of the tests found patients with folate deficiency, 166 (0.197%), found patients with low-normal folate levels, 57,411 (68.195%) of tests yielded normal results, and 26,563 (31.552%) of tests found high folate levels. The opinion of the authors was that folate testing should be severely reduced or eliminated. Furthermore, the American Society for Clinical Pathology, as part of the Choosing Wisely campaign, states: “Do not order red blood cell folate levels at all.”8

So what does this all mean? We have been taught to have a reflex response to the evaluation of macrocytosis to test for B12 and folate. Neither of these are particularly common causes of macrocytosis, and in countries where there is grain fortification, folate deficiency is exceedingly uncommon, and should not be tested for early in any diagnostic process.
 

Dr. Paauw is professor of medicine in the division of general internal medicine at the University of Washington, Seattle, and he serves as third-year medical student clerkship director at the University of Washington. He is a member of the editorial advisory board of Internal Medicine News. Dr. Paauw has no conflicts to disclose. Contact him at imnews@mdedge.com.

References

1. Seppä K et al. Evaluation of macrocytosis by general practitioners. J Stud Alcohol. 1996 Jan;57(1):97-100.

2. Seppä K et al. Blood count and hematologic morphology in nonanemic macrocytosis: Differences between alcohol abuse and pernicious anemia. Alcohol. 1993 Sep-Oct;10(5):343-7.

3. Wymer A, Becker DM. Recognition and evaluation of red blood cell macrocytosis in the primary care setting. J Gen Intern Med. 1990 May-Jun;5(3):192-7.

4. Davidson RJ, Hamilton PJ. High mean red cell volume: Its incidence and significance in routine haematology. J Clin Pathol. 1978 May;31[5]:493-8.

5. Joelson DW, Fiebig EW. Diminished need for folate measurements among indigent populations in the post folic acid supplementation era. Arch Pathol Lab Med. 2007 Mar;131(3):477-80.

6. Shojania AM, von Kuster K. Ordering folate assays is no longer justified for investigation of anemias, in folic acid fortified countries. BMC Res Notes. 2010 Jan 25;3:22. doi: 10.1186/1756-0500-3-22.

7. Theisen-Toupal et al. Low yield of outpatient serum folate testing. JAMA Intern Med. 2014 Oct. doi: 10.1001/jamainternmed.2014.3593.

8. Choosing Wisely: American Society for Clinical Pathology, Oct. 19, 2017. Recommendation.

Publications
Publications
Topics
Article Type
Sections
Disallow All Ads
Content Gating
No Gating (article Unlocked/Free)
Alternative CME
Disqus Comments
Default
Use ProPublica
Hide sidebar & use full width
render the right sidebar.
Conference Recap Checkbox
Not Conference Recap